Sei sulla pagina 1di 94

Copyright © 2014 Delhi Academy of Medical Sciences, All Rights Reserved.

1/94
Test Information
Test Name SWTS-ONCOLOGY-2017 (MDMS) Total Questions 200

Test Type Examination Difficulty Level Difficult

Total Marks 600 Duration 120minutes

Test Question Language:- ENGLISH

(1). Human papillomavirus (HPV) infection is associated with which type of cancer?

a. Cervical

b. Anogenital

c. Head and neck

d. All of the above

Solution. (d) All of the above


Ref: Read the text below.
Sol:
- Cervical cancer is a major cause of mortality in women worldwide and is initiated by infection with high-risk HPVs.
- These viruses, especially HPV-16, are associated with other anogenital cancers and a subgroup of head and neck cancers.
- Indeed, HPV infection could account for the development of head and neck cancer in persons who lack the classic risk factors for this
disease (tobacco and alcohol abuse

Correct Answer. d

(2). "Triple-negative" breast cancer has been defined by modern genomic techniques as which distinct breast tumor subset?

a. Luminal A

b. Luminal B

c. Basal-like

d. "Normal"-like

Solution. (a) Luminal A


Ref: Read the text below.
Sol:
- The use of modern genomic techniques has significantly enhanced our understanding of breast cancer biology.
- Five distinct breast cancer tumor subsets have been recognized, including hormone receptor (HR)-positive luminal A and B, human
epidermal growth receptor 2 (HER2)-positive, "normal"-like, and basal-like.
- This final group is frequently identified as "triple negative" by conventional immunohistochemicaltechniques because it lacks staining
for estrogen receptor, progesterone receptor, and HER2.
- Triple-negative tumors, which often overexpress the epidermal growth factor receptor 1 and are positive for CK 5 and/or 6, are typically
high grade and are associated with a high risk for relapse within the first several years after initial diagnosis.
- Long-term follow-up of triple-negative cohorts has demonstrated a worse prognosis for the triple-negative subgroups than for those that
are HR-positive

Correct Answer. a

Copyright © 2014 Delhi Academy of Medical Sciences, All Rights Reserved. 2/94
(3). Which type of lung cancer is exquisitely sensitive to chemotherapy ?

a. Squamous cell carcinoma

b. Adenocarcinoma

c. Large cell carcinoma

d. Small cell carcinoma

Solution. (d) Small cell carcinoma


Ref: Read the text below.
Sol:
- Any “non-small cell” carcinoma, including large cell carcinoma, squamous cell carcinoma, and adenocarcinoma, is relatively resistant to
chemotherapy.
- Small cell carcinoma is exquisitely sensitive to chemotherapy but invariably relapses in approximately 2 years or less.

Correct Answer. d

(4). Which of the following increases a woman’s risk of breast cancer?

a. Late first pregnancy

b. Long-term nursing

c. History of breast cancer in an aunt

d. Multiparity

Solution. (a) Late first pregnancy


Ref: Read the text below
Sol:
- A generally increased risk of breast cancer is associated with nulliparity, late first pregnancy,and, especially, a history of maternal
breast cancer.
- Prior history of breast cancer is of course a powerful risk factor

Correct Answer. a

(5). A27-year-old man with HIV complains of pain every time he swallows (odynophagia). He is not on any antiretroviral therapy and
otherwise feels well. Examination of the mouth and pharynx are normal. Which of the following is the most likely diagnosis?

a. Midesophageal cancer

b. Distal esophageal cancer

c. Candida infection

d. Bacterial esophagitis

Solution. (c) Candida infection


Ref: Read the text below
Sol:
- Painful swallowing can be caused by candida or herpes infection or pill-induced esophagitis.
- Patients with immunodeficiency states (e.g., AIDS) may have herpetic, candidal, or CMV esophagitis, as well as tumors (lymphoma,
Kaposi sarcoma)

Correct Answer. c

Copyright © 2014 Delhi Academy of Medical Sciences, All Rights Reserved. 3/94
(6). A42-year-old man received radiation exposure at a nuclear power plant. He is now concerned about his future cancer risk. Which of the
following statements is correct?

a. Malignancies occur within 10 years of exposure

b. Leukemia has the shortest latency period of all malignancies

c. Large exposure is required to develop the most serious malignancies

d. Risk increases with advancing age at the time of exposure

Solution. (b) Leukemia has the shortest latency period of all malignancies
Ref: Read the text below
Sol:
- Radiation-induced malignancies tend to occur at the age where that particular malignancy would normally occur.
- Therefore, the latency period can be 40 years or more. The latency period tends to be shortest (5–7 years) for leukemia.
- The risk for most malignancies is greatest with early-life radiation, and evidence suggests that therapeutic radiation confers excess risk
as well.
- The amount of exposure determines the likelihood of developing malignancy, not its severity.

Correct Answer. b

(7). Which of the following groups has an increased incidence of pancreatic cancer?

a. Caucasians

b. African-Americans

c. Males

d. Females

Solution. (b) African-Americans


Ref: Read the text below.
Sol:

Correct Answer. b

Copyright © 2014 Delhi Academy of Medical Sciences, All Rights Reserved. 4/94
(8). Treatment of hairy cell leukemia is :

a. Steroid

b. Multiple transfusion

c. Splenectomy

d. Cladribine

Solution. (d) Cladribine


Ref: Read the text below
Sol:
Hairy cell leukemia
Hairy cell leukemia, an uncommon form of leukemia is an indolent cancer of B lymphocytes.
Symptoms :-
ØMean age of presentation - 55 yrs.
ØMost patients present with gradual onset of fatigue, other symptoms are related to markedly enlarged spleen,some come to attention
because of infection.

Correct Answer. d

(9). Extended right hepatectomy / right lobectomy entails removal of following liver segments :-

a. II, III, IV, V, VI

b. III, IV, V, VI, VII

c. V, VI, VII, VIII, IX

d. V, VI, VII, VIII

Solution. (d) V, VI, VII, VIII


Ref: Read the text below
Sol:
· French anatomist couinaud has described the liver as being divided into eight segments.
· Each of these segments can be considered as functional units, each with a branch of hepatic artery, portal vein and bile duct and
drained by a branch of the hepatic vein. The overall anatomy of the liver is divided into a functional right and left lobe along the
line between the GB fossa and middle hepatic vein.
· Liver segments (V-VIII) to the right of this line are supplied by right hepatic artery and right branch of portal vein and drain bile
via right hepatic duct.
· To the left of this line (Segments I-IV) is left functional lobe. [Surgical lobes of liver compared with anatomical lobes of
liver]

Correct Answer. d

Copyright © 2014 Delhi Academy of Medical Sciences, All Rights Reserved. 5/94
(10) Which of the following adenomatous polyps have the greatest potential
. for malignant transformation ?

a. Villous adenomas

b. Tubovillous adenomas

c. Tubular adenomas

d. None of the above

Solution. (a) Villous adenomas


Ref: Read the text below.
Sol:

Correct Answer. a

(11). Which of the following is the most common site for squamous cell carcinoma of the esophagus to develop?

a. Upper esophageal sphincter

b. Lower esophageal sphincter

c. Proximal esophagus

d. Middle esophagus

Solution. (d) Middle esophagus


Ref: Read the text below.
Sol:

Correct Answer. d

Copyright © 2014 Delhi Academy of Medical Sciences, All Rights Reserved. 6/94
(12). The most common soft tissue sarcoma in children is:

a. Synovial sarcoma

b. Malignant fibrous histiocytoma

c. Fibrosarcoma

d. Rhabdomyosarcoma

Solution. (d) Rhabdomyosarcoma


Ref: Read the text below
Sol:
· Rhabdomyosarcoma is the most common soft tissue sarcoma in childhood.
· Rhabdomyosarcoma tumors arise from a cell called a "rhabdomyoblast", which is a primitive muscle cell. Instead of differentiating
into striated muscle cells, the rhabdomyoblasts grow out of control. Since this type of muscle is located throughout the body, the
tumors can appear at numerous locations. The four major sites in which rhabdomyosarcoma is found are:
· Head and neck; around the eyes -- 35-40%
· Genitourinary tract -- 20%
· Extremities -- 15-20%
· Trunk (chest and lungs) -- 10-15%
· Depending on the "histology" of the cells (how they look under a microscope), the tumors are classified as one of the following:
· Embryonal rhabdomyosarcoma. Most common type, usually found in children under 15 and in the head and neck region and
genitourinary tract.
· Botryoid type. A variant of the embryonal type; the tumor arises as a grape-like lesion in mucosal-lined hollow organs such as the
vagina and urinary bladder.
· Alveolar type. A more aggressive tumor which usually involves the muscles of the extremities or trunk.
· Pleomorphic type. Usually seen in adults and arises in muscles of the extremities.

Correct Answer. d

(13). All of the following statements about Gleason's grading system is true except:

a. High score is associated with poor prognosis

b. Helps in grading of tumor

c. Score ranges from 1 to 10

d. Helps decide treatment modality

Solution. (c) Score ranges from 1 to 10


Ref:Read the text below
Sol:
Gleason staging system
· Most popular grading system for prostate carcinoma
· Based on architectural rather than histologic criteria
· Grade 1 (well differentiated) – 5 (undifferentiated) are assigned which correlates with tumor volume, stage & prognosis
· Most poorly differentiated area determines biologic behavior. Perineural & extracapsular spread are also recorded
· Primary grade is applied to architectural pattern to architectural pattern of cancerous gland occupying largest area of specimen
while secondary pattern is assigned to next largest area of cancerous growth
· Adding primary & secondary pattern gives Gleason score (2-10) for each tumor.

Correct Answer. c

Copyright © 2014 Delhi Academy of Medical Sciences, All Rights Reserved. 7/94
(14). True about pancreatic carcinoma is all except:

a. Deactivation of p16

b. Presence of panIN's

c. Altered NRAS

d. Inactivation of p53

Solution. (c) Altered NRAS


Ref:Read the text below
Sol:
Pancreatic cancer:
· K-RAS is most frequently altered oncogene
· p16 is most frequently inactivated tumor suppressor gene followed by p53, SMAD4
· Associated with pancreatic intra-epithelial neoplasms (panIN’s), that serve as pre-malignant lesions
· Site: most common site is head (60%) followed by body (15%) and tail (5%)
· Morphology:
✔ Hard, stellate, grey-white, poorly defined mass
✔ Characteristic feature is its high invasiveness and an intense non-neoplastic host reaction composed of fibroblast, lymphocytes, &
extracellular matrix called desmoplastic reaction
· Most common symptom- abdominal pain
· Most common physical sign- jaundice
· Trousseau sign- migratory thrombophlebitis
· Diagnosis:
ØTumor marker: CEA & CA19-9 (most useful) are most consistently used tumor markers
ØImaging: Barium meal- in Ca head pancreas, there is widening of C-loop of duodenum celled pad sign
ØEndoscopic ultrasonography- imaging modality of choice for diagnosis
ØMultidetector CT with dual phase imaging- modality of choice for staging of pancreatic cancer

Correct Answer. c

(15). True about Fibrolamellar carcinoma are all except:

a. Affects young age group with equal sex incidence

b. Schirrous presentation

c. Always associated with underlying chronic liver disease

d. Cells arranged in nests & chords

Solution. (c) Always associated with underlying chronic liver disease


Ref:Read the text below
Sol:
Fibrolamellar variant of HCC:
o Seen in young adults (20-40 yrs)
o Equal incidence in males and females (in India, however, females > males)
o No association with HBV or cirrhosis risk factors (no association with chronic liver disease)
o Better prognosis
o No elevation of AFP levels
o Microscopy shows well differentiated cells arranged in nests & chords separated by dense collagen bands
o Fibrolamellar variant usually affects left lobe more commonly
o Spreads by lymphatic route
o Tumor marker- NEUROTENSIN
o Marker of choice for differentiating between HCC & its fibrolamellar variant is AFP

Correct Answer. c

Copyright © 2014 Delhi Academy of Medical Sciences, All Rights Reserved. 8/94
(16). Gastric MALTomas exhibit which of the following features:

a. Inactivation of NF-KB

b. Positivity for CD43

c. Absence of lymphoepithelial lesions

d. All of above

Solution. (b) Positivity for CD43


Ref:Read the text below
Sol:
Extra-nodal marginal zone B-cell lymphomas (MALTomas):
· Usually arise at sites of pre-existing chronic inflammation
· Most common cause of “pro-lymphomatous” inflammation in stomach is chronic H.pylori infection, which is found in association
with most cases of gastric MALToma
· 3 translocations are associated with gastric MALToma- t(11:18), t(14:18), t(1:14)
· NF-ⱪβ is constitutively active intumors bearing translocations involving MLT or BCL10, & as a result the elimination of H.pylori
has no effect
· Additional genetic changes, such as inactivation of tumor suppressor genes that encode p53 & p16, may lead to transformation of
gastric MALToma into aggressive diffuse large B-cell lymphoma
· Morphology:
· Characteristically, the neoplastic lymphocytes infiltrate the gastric glands focally to create diagnostic lymphoepithelial lesions
· Express CD19, CD20
· Positive for CD43 in about 25% of cases, an unusual feature that can be diagnostically helpful

Correct Answer. b

(17). Biologic features of adult soft tissue sarcomas include the following except:

a. Recurrent disease in atleast 33% of patients

b. Lymph node metastasis in less than 3% of patients

c. Mutation of p53 in metastatic liposarcoma

d. A low (less than 1%) risk of metastasis for small, low-grade lesions

Solution. (c) Mutation of p53 in metastatic liposarcoma


Ref:Read the text below
Sol:
Soft tissue sarcomas:
· Inactivation of p53 is involved in tumorigenesis of several sarcomas, but not specifically liposarcoma
· Low-grade lesions have a low (less than 1%) risk of subsequent metastasis, whereas high-grade lesions have a high (greater than
50%) risk of subsequent metastasis
· Despite optimal multimodality treatment, at least 33% of patients develop recurrent disease
· Lymph node metastasis occur in fewer than 3% of adult soft tissue sarcomas

Correct Answer. c

Copyright © 2014 Delhi Academy of Medical Sciences, All Rights Reserved. 9/94
(18). Cytotoxic T cells (CTL) are capable of recognizing:

a. Peptide antigens associated with major histocompatibility complex (MHC) molecules.

b. Membrane-bound antigens.

c. Cytoplasmic antigens.

d. All of the above.

Solution. (d) All of the above


Reference – Read the text below
Sol
· It was long thought that CTL recognizeantigen expressed on the surface of the target cell destined to be killed, similar to the
recognition of an antigen by an antibody; however, it was found that the mechanism of CTL recognition of antigens was
fundamentally different from the mechanism of antibody recognition.
· CTL can detect antigensderived from cell surface–associated proteins, but in addition can recognize proteins that are normally in
the cytoplasm or in the nucleus. In fact, the normal location of the protein can be anywhere within the cell.
· To understand how a CTL can distinguish an antigenfrom a protein that is normally located within the cytoplasm or nucleus of
the cell, investigators had to determine exactly what was being recognized by the CTL. T cells were found to recognize short linear
fragments of processed or even denatured protein.
· It was found that proteins that were synthesized endogenously were degraded within the cytoplasm into 9– or 10–amino acid long
peptides. These peptides were then transported to the endocytoplasmic reticulum and associated with newly synthesized MHC
class I molecules.
· Certain peptides could fit within the MHC class I moleculeand were then transported to the cell surface as a complex. It was
this complex, consisting of a 9– to 10–amino acid peptide within a MHC class I molecule that was presented to, and recognized by,
CD8+ CTL.

Correct Answer. d

(19). Cislatin

a. Is devoid of renal toxicity

b. Is ineffective in ovarian cancer

c. Is effective in advanced testicular carcinoma

d. Acts by inhibiting DNA dependent RNA polymerase.

Solution. (c) Is effective in advanced testicular carcinoma


Reference – Read the text below
Sol
· Cisplatin, a platinum coordination complex, is most useful in testicular and ovarian carcinoma.
· It acts by forming a highly reactive moiety after intracellular hydrolysis, which cause cross linking of DNA.
· The most important toxicity of cisplatin is renal toxicity. It can be minimized by adequate hydration by giving i.v. fluids.
· Also, it causes tinnitus, deafness and peripheral neuropathy. Therefore, cisplatin is oto-nephro-neuro-toxic (like
aminoglycosides).
· Cisplatin is in disrepute as it causes maximum vomiting. Also, it causes anaphylactoid reactions, electrolyte disturbances and
hyperuicemia.

Correct Answer. c

Copyright © 2014 Delhi Academy of Medical Sciences, All Rights Reserved. 10/94
(20). Virus associated with hepatocellular pathology to produce jaundice is:

a. Hepatitis virus A, B, C, D and E

b. Cytomeglovirus

c. Epstein-Barr virus

d. All of the above.

Solution. (d) All of the above.


Reference – Read the text below
Sol
There are many viruses which affect the liver and produce jaundice.
They are :
(a) Hepatitis virus A, B, C, D, E
(b) Cytomegalovirus
(c) Epstein – Barr virus
(d) Herpes Simplex.

Correct Answer. d

(21). Adoptive immunotherapy with lymphokine activated killer cells (LAK) and tumor infiltrate (TIL) cells are characterized by:

a. Nonspecific stimulation of effector cells.

b. Expansion ex vivo of large numbers of lymphocytes.

c. Infusion with interleukin 2 (IL-2).

d. All of the above

Solution. (d) All of the above


Reference – Read the text below
Sol
· LAK cells, generated by short-term culture of peripheral blood lymphocytesin the presence of high concentrations of IL-2,
lyse transformed target cells, and have minimal lytic activity for most normal tissues.
· Up to 10 11 in vitro generated LAK cells havebeen administered in a single intravenous infusion to cancer patients. Therapeutic
trials have also combined short courses of high-dose systemic IL-2 administration with LAK cell transfer to promote LAK function
and viability, with apparently enhanced efficacy.
· The shortcomings of LAK and IL-2 therapy included a larger degree of toxicity (including pulmonary, renal, and hepatobiliary) with
a significant proportion of patients requiring intensive care unit admissions and 2% to 5% treatment-related mortality.
Despite this, response rate remained relatively low.
· Another therapy usingin vitro expanded lymphocytes derived from a TIL has been evaluated in clinical trials.
· In humans, TILcell lines have been generated by mincing tumor specimens and culturing eluted lymphocytes with high
concentrations of IL-2. TIL lines can be expanded to 10 8 to 10 11 cells over 3 to 8 weeks in culture, and some lines appear to function as
T cells with lytic specificity for autologous—but not allogeneic—tumor targets, whereas others function as LAK cells and lyse both
autologous and allogeneic tumor targets.
· Adoptive transfer of 5 × 10 10 TILalone has not been associated with significant toxicity, and administration of 5 × 10 10 TIL
cells with concurrent systemic IL-2 has caused toxicities that are attributable to the IL-2.

Correct Answer. d

Copyright © 2014 Delhi Academy of Medical Sciences, All Rights Reserved. 11/94
(22). Previous clinical studies with cancer vaccines have:

a. Clearly demonstrated induction of tumor-specific immune response.

b. Repeatedly demonstrated clinical response to large tumor burden.

c. Not clearly demonstrated induction of tumor-specific immune response.

d. Not been performed to date.

Solution. (c) Not clearly demonstrated induction of tumor-specific immune response


Reference – Read the text below
Sol
· Several different approaches in experimental animal studies in the 1970s formed the basis for human clinical trials of SAI
against human cancers.
· Although several thousand patients have been injected with a variety of tumor cell preparations in this country and elsewhere
during the past 25 years, the complexity of the studies has made it difficult to definitively assess the value of this approach to cancer
therapy.
· Nonetheless, a number of clinical trials have suggested a therapeutic benefit of SAI.
· Most of these trials have two features in common: (1) a therapeutic effect was not seen or, if seen, has not been confirmed
independently; and (2) no acceptable information was provided on the presence of tumor antigens in the vaccines and the immune
response of patients to these antigens.

Correct Answer. c

(23)
Storiform pattern is seen in?
.

a. Synovial sarcoma

b. Myxoid liposarcoma

c. Schwann cell tumor

d. Fibrohistiocytic tumor

Solution. (d) Fibrohistiocytic tumor


Ref:Read the text below
Sol:

Correct Answer. d

Copyright © 2014 Delhi Academy of Medical Sciences, All Rights Reserved. 12/94
(24). True about osteoclastoma are all except:

a. Present in age group 20-40 yrs

b. Use RANK/RANKL signaling pathway

c. Aneurysmal bone cyst common D/D

d. Osteoclast-like giant cells constitute the proliferating component of tumor

Solution. (d) Osteoclast-like giant cells constitute the proliferating component of tumor
Ref:Read the text below
Sol:
Giant cell tumor (osteoclastoma):
· Mixture of mononuclear cells & a profusion of multinucleated osteoclast-type giant cells
· Usually arises in individuals in their 20s to 40s
· Relatively uncommon benign but locally aggressive neoplasm
· Mononuclear cells in giant cell tumor express RANKL & giant osteoclast-type giant cells are believed to form via the RANK/RANKL
signaling pathway
· Morphology:
✔ Mostly composed of uniform oval mononuclear cells that constitute the proliferating component of the tumor
✔ Scattered within background are numerous osteoclast-type giant cells having 100 or more nuclei that resemble those of mononuclear
cells
· Site:
✔ GCTs in adults involve both the epiphyses & metaphyses, but in adolescents they are confined proximally by the growth plate & are
limited to metaphysic
✔ Majority arise around knee (distal femur & proximal tibia), but virtually any bone can be involved
· Clinical features:
o Typical location of these tumors around joints gives rise to arthritis like symptoms
o Occasionally present with pathologic fractures
o Most are solitary, however, multiple or multicentric tumors do occur, especially in distal extremities

Correct Answer. d

(25). What is the best treatment approach for aggressive nonmelanoma skin cancer (NMSC) when disfigurement or functional impairment is a
risk?

a. Mohs micrographic surgery

b. Radiation therapy

c. Imiquimod

d. Photodynamic therapy

Solution. (a) Mohs micrographic surgery


Ref: Read the text below.
Sol:
- Surgery is the most frequent approach used to treat NMSC. The gold standard for treatment continues to be Mohs micrographic
surgery, but owing to the time and expense involved with this procedure, it is indicated only in patients with aggressive tumors or those
for whom disfigurement or functional impairment is a risk.
- Although radiation therapy is effective, its use is limited because of the side effects, but it can be used in certain patients who are not
surgical candidates. Imiquimod is a topical immune-response modulator that is effective against superficial basal cell carcinoma (BCC),
small nodular BCC, and squamous cell carcinoma (SCC).
- Photodynamic therapy involves application of a photosensitizing compound to the skin that preferentially accumulates within the tumor
cell and is then activated by a light source.
- Photodynamic therapy is effective in treating SCC in situ and superficial BCC

Correct Answer. a

Copyright © 2014 Delhi Academy of Medical Sciences, All Rights Reserved. 13/94
(26). What is a contraindication to the use of bevacizumab in patients with metastatic non-small-cell lung cancer (NSCLC)?

a. Previous hemoptysis

b. Brain metastases

c. Anticoagulation

d. All of the above

Solution. (d) All of the above


Ref: Read the text below.
Sol:
- Sandler and colleagues reported a survival benefit with the addition of bevacizumab to carboplatin and paclitaxel, a finding that
represents the first time median survival has been extended beyond 1 year in advanced NSCLC.
- Patients in the bevacizumab group demonstrated a significantly higher response rate, improved progression-free survival, and better
median overall survival than the chemotherapy-alone arm.
- The researchers concluded that carboplatin/paclitaxel in combination with bevacizumab should be the new standard first-line approach
for patients with nonsquamous NSCLC, no previous hemoptysis, and no brain metastases and for those not receiving therapeutic
anticoagulation.

Correct Answer. d

(27). Which of the following chemopreventive agents has been shown to lower prostate cancer occurrence?

a. Finasteride

b. Lycopene

c. Selenium

d. All of the above

Solution. (a) Finasteride


Ref: Read the text below.
Sol:
- The Prostate Cancer Prevention Trial (PCPT) was the first prospective chemoprevention trial of finasteride to demonstrate a reduction
in the primary outcome of prostate cancer, although the risk for high-grade tumors was elevated.
- Intake of foods high in vitamin E, lycopene, or selenium has also been associated with lower prostate cancer risk in epidemiologic and
retrospective studies.
- Data from nested case-control studies indicate that a beneficial effect from beta-carotene and selenium may be restricted to men with
low baseline plasma levels.
- Although some of the data seem promising, at this point it is premature to recommend nutritional intervention as a way to prevent
prostate cancer

Correct Answer. a

(28). Dysembryoplastic neuroepithelial tumor is characterized by:

a. Rare adult tumor

b. High grade lesion

c. May present as a seizure disorder

d. All of above

Solution. (c) May present as a seizure disorder


Ref:Read the text below
Sol:
Dysembryoplastic neuroepithelial tumor
· Rare, low-grade tumor of childhood that often presents as a seizure disorder
· Has a relatively good prognosis after surgical excision
· Typically located in superficial temporal lobe
· Morphology:
üThese lesions typically form multiple discrete intracortical nodules of small round cells, arranged in columns around central core of
processes, & are associated with a myxoid background known as the “specific glioneuronal element”

Correct Answer. c

Copyright © 2014 Delhi Academy of Medical Sciences, All Rights Reserved. 14/94
(29). True about glioblastoma are all except:

a. Inactivation of p16

b. Glomeruloid body

c. Pseudopalisading

d. Suppression of hepatocyte growth factor -scatter factor

Solution. (d) Suppression of hepatocyte growth factor –scatter factor


Ref:Read the text below
Sol:
Glioblastoma (glioblastoma multiforme)
· Variation in gross appearance from region to region is characteristic
· In addition to features of anaplastic astrocytoma, necrosis & endothelial cell proliferation also seen
· Necrosis in glioblastoma often occurs in a serpentine pattern in areas of hypercellularity. Tumor cells collect along edges of the
necrotic regions, producing a histologic pattern referred to as pseudopalisading
· Vascular cell proliferation is characterized by tufts of piled-up cells that bulge into the lumen; the minimum criterion for this
feature is a double layer of endothelial cells. With marked vascular proliferation the tuft forms a ball-like structure, the glomeruloid
body
· Inactivation of Rb & p16/CDKN1A
· Secondary glioblastomas usually have p53 mutations, primary glioblastomas more commonly have amplification of MDM2

Correct Answer. d

(30). RAEB-2 most commonly progress to

a. CLL

b. ALL

c. AML

d. CML

Solution. (c) AML .


Reference – Read the text below
Sol
- RAEB-2 (refractory anaemia with excessive blasts-2) comes under WHO group of MDS (myelodysplastic syndrome).
- It usually progresses to Acute myeloid leukemia (AML)

Correct Answer. c

Copyright © 2014 Delhi Academy of Medical Sciences, All Rights Reserved. 15/94
(31). Cell shown in Fig is seen in

a. CLL

b. ALL

c. AML

d. CML

Solution. (d) CML.


Reference – Read the text below
Sol

Correct Answer. d

(32). Crohn’s disease patients treated with infliximab usually develop which lymphoma?

a. Enteropathic T cell lymphoma / enteropathy associated T cell lymphoma

b. Panniculitis like T cell lymphoma

c. ALCL

d. Hepatosplenic T cell lymphoma

Solution. (d) Hepatosplenic T cell lymphoma.


Reference – Read the text below
Sol
- Crohn’s disease patients when treated with infliximab and azathioprine, usually develop hepatosplenic T cell lymphoma

Correct Answer. d

Copyright © 2014 Delhi Academy of Medical Sciences, All Rights Reserved. 16/94
(33). Which of the following NHL usually express CD30, a marker of Hodgkin’s lymphoma

a. DLBCL

b. Follicular lymphoma

c. ALCL

d. Adult T cell lymphoma / leukemia

Solution. (c) ALCL


Reference – Read the text below
Sol
- ALCL (Anaplastic large cell lymphoma) usually express CD30.
- CD30 is classically a marker of Hodgkin’s lymphoma.
- In spite of being a NHL (non-Hodgkins lymphoma), ALCL expresses CD30

Correct Answer. c

(34). Which one of the following marker is most useful to diagnose M6 variant of AML

a. CD41

b. CD61

c. Glycophorin A

d. SBB

Solution. (c) Glycophorin A.


Reference – Read the text below
Sol
- M6 variant of AML shows increased number of erythroblasts.
- They are usually positive for Glycophorin A

Correct Answer. c

(35). Which of the following variables best predicts prognosis for patients with a recent diagnosis of cutaneous melanoma and no clinical
evidence of metastatic disease?

a. Breslow thickness.

b. Clark's level.

c. Ulceration.

d. Gender.

Solution. (a) Breslow thickness


Reference – Read the text below
Sol
- The prognosis for melanoma is best predicted by the thickness, measured in millimeters (Breslow thickness).
- Clark's level and ulceration are also predictive, but less so. Gender is a secondary prognostic factor.
- Skin color may have a mild impact on outcome, but it is primarily a risk factor for developing melanoma.

Correct Answer. a

Copyright © 2014 Delhi Academy of Medical Sciences, All Rights Reserved. 17/94
(36). Seminoma shows the following features except:

a. Negative staining for c-KIT

b. Secrete HCG

c. Prominent eosinophilic nucleolus

d. Tigroid background

Solution. (a) Negative staining for c-KIT


Ref:Read the text below
Sol:
Seminoma
- Seminomatous cells are diffusely positive for c-KIT, OCT4, & placental alkaline phosphatase
- HCG elevation commonly seen (non-seminomatous germ cell tumors express AFP)
- Typical seminoma has a homogenous, gray white loculated appearance, usually devoid of necrosis, hemorrhage
- Classical seminoma cell is large & round to polyhedral & has a distinct cell membrane; a clear or watery-appearing cytoplasm; and a
large central nucleus with prominent eosinophilic nucleolus
- Cytologically, seminoma shows polyhedral cells in a background of scattered lymphocytes and loose fibrillary element, referred to as
tigroid appearance

Correct Answer. a

(37). Of the following, which is not a feature of PSTT:

a. May be preceded by hydatidiform mole

b. Proliferation of extravillous trophoblast

c. Elevation of AFP

d. Proliferation of intermediate trophoblast

Solution. (c) Elevation of AFP


Ref:Read the text below
Sol:
Placental-site trophoblastic tumor (PSTT)
- Compose less than 2% of gestational trophoblastic tumors
- Represent neoplastic proliferation of extravillous trophoblast, also called intermediate trophoblast
- Malignant transformation of intermediate trophoblast gives rise to PSTT, which presents as a uterine mass, accompanied by either
abnormal uterine bleeding or amenorrhoea& moderate elevation of β-HCG.
- Histologically, PSTT is composed of malignant trophoblastic cells diffusely infiltrating the endomyometrium
- PSTTs may be preceded by a normal pregnancy (one half), spontaneous abortion (one sixth), or hydatidiform mole (one fifth)
- In normal pregnancy, extravillous (intermediate) trophoblast is seen in nonvillous sites such as implantation site, in islands of cells
within placental parenchyma, in the chorionic plate, & in the placental membranes

Correct Answer. c

Copyright © 2014 Delhi Academy of Medical Sciences, All Rights Reserved. 18/94
(38). Which of the following statement about Androblastomas is true:

a. Bilateral tumors

b. May show heterologous elements

c. Cause feminization

d. Peak incidence in sixth decade

Solution. (b) May show heterologous elements


Ref:Read the text below
Sol:
Sertoli-leydig cell tumors (Androblastomas)
- Commonly produce masculinization or at least defeminization, but a few have estrogenic effets
- Occur in women of all ages, although the peak incidence is in the second & third decades
- Unilateral tumors
- On histologic examination, tumors show tubules composed of sertoli cells or leydig cells interspersed with stroma
- Poorly differentiated tumors have a sarcomatous pattern with a disorderly deposition of epithelial cell cords. Leydig cells may be
absent. Heterologous elements, such as mucinous glands, bone, & cartilage, may be present in some tumors

Correct Answer. b

(39). The following is an Electron microscopic image of an ileal mass in patient with diarrhoea. What is likely diagnosis?

a. Carcinoid tumor

b. Adenocarcinoma

c. Lymphoma

d. Langerhan cell histiocytosis

Solution. (a) Carcinoid tumor


Ref: Read the text below.
Sol:
- This EM shows dense core neurosecretory granules which is seen in neuroendocrine tumors.

Correct Answer. a

Copyright © 2014 Delhi Academy of Medical Sciences, All Rights Reserved. 19/94
(40). All of the following soft tissue sarcomas have propensity for lymphatic spread except:

a. Rhabdomyosarcoma

b. Synovial sarcoma

c. MFH

d. Epitheloid sarcoma

Solution. (c) MFH


Ref:Read the text below
Sol:
Lymphatic spread in soft tissue sarcomas is seen in:
1. Rhabdomyosarcoma
2. Synovial sarcoma
3. Epithelioid sarcoma
4. Clear cell sarcoma
5. Angiosarcoma
6. Fibrosarcoma (rare)
Highest incidence of lymphatic metastasis is seen in embryonal rhabdomyosarcoma Routine prophylactic node dissection is not indicated
even in these tumors Node dissection is done only if the regional nodes are enlarged or the tumor is very close to the draining nodes

Correct Answer. c

(41). All of the following are signs of inoperability for carcinoma stomach except:

a. Ascites

b. Omental involvement

c. Liver metastasis

d. Peritoneal deposits

Solution. (b) Omental involvement


Ref:Read the text below
Sol:
- Omentectomy is done in gastrectomy for carcinoma stomach to clear the nodes along the greater curvature

Correct Answer. b

Copyright © 2014 Delhi Academy of Medical Sciences, All Rights Reserved. 20/94
(42) All of the following are good prognostic factors for germ cell tumors
. except:

a. Absence of para-aortic nodes

b. Seminomatous histology

c. Pulmonary metastasis

d. High levels of serum LDH

Solution. (d) High levels of serum LDH


Ref:Read the text below
Sol:
Testicular tumors
- Higher the level of tumor markers, poorer is the prognosis in testicular
tumors
- Stage is the most important prognostic factor
- Other prognostic factors are-
- Histology; seminomas carry better prognosis
- Site of distant metastasis- pulmonary metastasis carry good prognosis
as they are curable with lung resection

LDH: Elevated in all seminomatous& non-seminomatous tumors Reflects


tumor burden High values are associated with poor prognosis

Correct Answer. d

Copyright © 2014 Delhi Academy of Medical Sciences, All Rights Reserved. 21/94
(43) Sentinel node concept is established in all of the following tumors
. except:

a. Melanoma

b. Breast carcinoma

c. Skin appendageal tumor

d. Oral carcinoma

Solution. (d) Oral carcinoma


Ref:Read the text below
Sol:

Correct Answer. d

Copyright © 2014 Delhi Academy of Medical Sciences, All Rights Reserved. 22/94
(44). “Barrett’s epithelium”represent?

a. Cellular hyperplasia

b. Cellular hypertrophy

c. Metaplasia

d. Carcinoma in situ

Solution. (c) Metaplasia


Reference – Read the text below
Sol
- Hyperplasia is an increase in cell number. For example, an increase in growth rate over baseline is a normal response in wound healing.
- Liver regeneration after hepatic injury or hepatectomy results in both a dramatic hyperplastic response with an increase in cell number
and a hypertrophic response with an increase in cell size.
- Metaplasia is a reversible transformation of one mature cell type to another in a given tissue. Epithelial metaplasia secondary to chronic
inflammation has been extensively investigated.
- In cases of chronic gastroesophageal reflux, the normal distal esophageal squamous cell mucosa may undergo metaplasia to a “gastric-
type” columnar epithelium.
- In the respiratory tract, the normal pseudostratified columnar epithelium may be replaced by squamous cells. Dysplasia is a term
usually applied to epithelial tissues, characterized by altered cell size, shape, and organization.
- Dysplasia is classified as mild, moderate, or severe, depending on the degree of cellular dedifferentiation. The gastrointestinal tract,
respiratory tree, urinary bladder, cervix, vagina, breast, and the skin all may exhibit foci of dysplasia. Nuclear polymorphism and
hyperchromatism are present and often accompanied by total loss of cellular and epithelial polarity. Mitoses are more frequently seen in
the afflicted areas than in the surrounding normal epithelium.
- Although disordered growth of superficial tissue is present, there is no penetration of abnormal cells through the epithelial basement
membrane.
- Carcinoma in situ demonstrates all of the above changes, with an increased number of mitoses, but it is still confined by the basement
membrane and may be considered the most disordered extreme of dysplasia.

Correct Answer. c

(45). Which anti-emetic is most effective >24 post chemotherapy:

a. Dexamethasone

b. Ondansetron

c. Tetracanniboids

d. Maxolon

Solution. (b) Ondansetron


Reference – Read the text below
Sol
- Nausea and vomiting are major side effects of cancer chemotherapy.
- Progress in preventing and treating chemotherapy-induced nausea and vomiting has resulted from the development of newer and more
effective antiemetics, the completion of prospective clinical trials that addressed the problem, and the education of physicians regarding
prevention and management.
- The serotonin antagonists ondansetron and granisetron are the newest and among the most effective antiemetic agents. They
selectively block the serotonin receptor 5-HT3, which is present peripherally on the vagus nerve and centrally in the chemoreceptor
trigger zone.
- They are effective in the treatment of nausea and vomiting due to cisplatin but are without the dystonic reactions that may accompany
metoclopramide.

Correct Answer. b

Copyright © 2014 Delhi Academy of Medical Sciences, All Rights Reserved. 23/94
(46). A 65yr old male presents with persistent cough and chest pain. On chest X-ray there is a 4x4cm coin lesion abutting the chest wall. The
investigation of choice is:

a. Bronchoscopic biopsy

b. Transthoracic FNAC

c. CT scan

d. PET scan

Solution. (b) Transthoracic FNAC


Ref:Read the text below
Sol:
o Transthoracic FNAC is the investigation of choice for peripheral lung tumors (adenocarcinoma & large cell carcinoma) whereas
Bronchoscopy is the investigation of choice for central lung cancer (squamous cell carcinoma).
o PET-CT or CT scan is the staging investigation of choice

Correct Answer. b

(47). All of the following are features of epithelial ovarian cancers except:

a. Majority present in stage III & IV

b. Debulking surgery recommended for advanced cancers

c. Intraperitoneal chemotherapy can be used

d. Whole abdominal radiation preferred over chemotherapy for lesser toxicity in advanced cancers

Solution. (d) Whole abdominal radiation preferred over chemotherapy for lesser toxicity in advanced cancers
Ref:Read the text below
Sol:
Ovarian tumors
- Currently there is no role of RT in the primary treatment of ovarian cancers because chemotherapy is equally effective as RT but carries
lesser toxicity
- In ovarian cancers intraepithelial cisplatin is as effective as intravenous cisplatin
- Intraperitoneal chemotherapy is administered by placing a chemoport or a Tenchoff’s catheter in the peritoneal cavity. Many patients
have poor tolerance to intraperitoneal chemotherapy & therefore intraperitoneal chemotherapy is not universally recommended.
STAGING FOR OVARIAN CANCER:
Stage I: Growth limited to ovaries
Ia- one ovary involved
1b- both ovaries involved
Ic- Ia or Ib& ovarian surface tumor, ruptured capsule, malignant ascites, or peritoneal cytology positive for malignant cells Stage
II: Extension of the neoplasm from ovary to pelvis
IIa- extension to uterus or fallopian tube
IIb- extension to other pelvic tissues
IIc- IIa or IIb& ovarian surface tumor, ruptured capsule, malignant ascites, or peritoneal cytology positive for malignant cells
Stage III: Disease extension to the abdominal cavity
IIIa- abdominal peritoneal surfaces with microscopic metastasis
IIIb- tumor metastasis <2cm in size
IIIc- tumor metastasis >2cm in size or metastatic disease in the pelvic, para-aortic, or inguinal lymph nodes Stage
IV: Distant metastatic disease
Malignant pleural effusion
Pulmonary parenchymal metastasis
Liver or splenic parenchymal metastasis (not surface implants)
Metastasis to supraclavicular nodes or skin
TUMOR STAGE IS THE MOST IMPORTANT PROGNOSTIC FACTOR

Correct Answer. d

Copyright © 2014 Delhi Academy of Medical Sciences, All Rights Reserved. 24/94
(48). The worst prognosis in renal cell carcinoma is seen with the following histological variant:

a. Tall cell

b. Clear cell

c. Papillary

d. Sarcomatoid

Solution. (d) Sarcomatoid


Ref:Read the text below
Sol:
- Type I Papillary &Chromophobevariants of renal cell carcinoma carry best prognosis
- Sarcomatoid variant having the highest nuclear grade & showing presence of diffuse sarcomatoid transformation in stroma (diffuse
fibroblastic proliferation having anaplastic nuclei & high mitotic activity) carries the worst prognosis

Correct Answer. d

(49). All of the following are true about Germ cell tumors of testis except:

a. Seminoma is the commonest histology

b. Seminomatous histology with elevated AFP should be treated as nonseminomatous tumors

c. Can undergo teratomatous change after completion of chemotherapy

d. Pulmonary metastasis are incurable

Solution. (d) Pulmonary metastasis are incurable


Ref:Read the text below
Sol:
- Pure seminomas will never have elevated AFP & if AFP is elevated, patient should be treated like NSGCT
- After chemotherapy, any residual para-aortic mass >3cm should be treated with retroperitoneal lymph node dissection (RPLND) as
these tumors can undergo teratomatous change after chemotherapy & the residual disease can be a teratoma.
- Pulmonary metastasis can be treated by lung resection

Correct Answer. d

(50). Preoperative radiotherapy used in all of the following cancers except:

a. Rectum

b. Esophagus

c. Colon

d. Cervix

Solution. (c) Colon


Ref:Read the text below
Sol:
- Routine pre-operative radiotherapy is recommended only in rectal cancers.
- Also sometimes used in stage Ib&IIa (bulky >4cm) carcinoma cervix and carcinoma oesophagus

Correct Answer. c

Copyright © 2014 Delhi Academy of Medical Sciences, All Rights Reserved. 25/94
(51). A 10 year boy was brought to hospital with complains of bloody urine. Urine microscopy shows RBC and RBC casts in urine. His maternal
grandfather had deafness and died of renal failure. One of his maternal cousins also has similar manifestations. The family pedigree is as
shown below. Which of the following is not true regarding this case

a. He will not transmit the disease to his sons

b. Basket weave appearance on electron microscopy

c. Renal transplant is the treatment

d. Frequent Recurrence even after transplant.

Solution. (d) Frequent Recurrence even after transplant.


Ref: Read the text below.
Sol:
- Frequent recurrence even after transplant.
- The diagnosis in this case is Alport syndrome, which is XLR disease so doesnot transmit from father to sons.
- EM shows basket weave appearance. The disease doesnot recur after renal transplant.

Correct Answer. d

Copyright © 2014 Delhi Academy of Medical Sciences, All Rights Reserved. 26/94
(52). All of the following are true about surgical margins in cancers except:

a. Skin melanoma margin of 1-3cm is dependent on depth of penetration

b. Sarcoma margin of 3-5cm is dependent on histology of the lesion

c. Submucosal spread in esophageal and stomach cancers is usually proximal

d. Circumferential clearance is sometimes more important in some cancers like rectal cancers

Solution. (b) Sarcoma margin of 3-5cm is dependent on histology of the lesion


Ref:Read the text below
Sol:
- Margin in a sarcoma is dependent on the site of the lesion and the grade of the lesion (grade is most important prognostic factor in
sarcomas)
- Circumferential clearance in rectal cancers is called Total Mesorectal Excision (TME) & is most important than distal margin to prevent
local recurrence
MELANOMA:
- Cell of origin- melanocyte
- Types:
1. Superficial spreading- commonest
2. Lentigomaligna melanoma (Hutchison’s freckle); least common & best prognosis
3. Nodular- highly malignant & affects younger patients. Prognosis poor
4. Acral-lentiginous- highly malignant & occur over palms & soles
5. Amelanotic- delayed diagnosis
- Signs of malignancy: change in size, outline, color, elevation, itching, bleeding in a mole, surrounding structure invasion, regional nodal
or distant metastasis
- Tissue diagnosis- Excision biopsy if possible
- Spread- lymphatic & blood borne (unusual sites metastasis common eg. , heart, adrenals)
- Staging:
- Clark’s staging- lateral spread
- Breslow’s staging- depth of tumor invasion
- AJCC staging- depth of tumor invasion, nodal & distant metastasis
- Treatment:
- Wide local excision & margin depends on depth of tumor invasion-
o <0.75mm- 1cm
o 0.75-1.5mm- 2cm
o .1.5mm- 3cm
- Regional nodal dissection- if nodes involved
- Prophylactic nodal dissection- controversial but mainly indicated in tumors of 1-4mm depth invasion. Sentinel node concept well
established in melanomas
- Treatment of metastasis- palliative surgical resection.

Correct Answer. b

(53). All of the following are indications for RT after limb salvage in soft tissue sarcomas except:

a. MFH histology

b. High grade tumors

c. Surgical margin 0.5cm

d. Size of tumor 6cm

Solution. (a) MFH histology


Ref:Read the text below
Sol:
- MFH is usually a low grade tumor therefore RT is not required after limb salvage unless other indications are present
- The indications for RT after limb salvage:
- Size of tumor >5cm
- High grade tumors
- Tumor margin positive or close (<1cm margin is close margin)
- Recurrent tumors

Correct Answer. a

Copyright © 2014 Delhi Academy of Medical Sciences, All Rights Reserved. 27/94
(54). Chemosensitive soft tissue sarcomas are all except:

a. Synovial sarcoma

b. PNET

c. Embryonal RMS

d. Fibrosarcoma

Solution. (d) Fibrosarcoma


Ref:Read the text below
Sol:
- Chemosensitive sarcomas are Ewing’s, PNET, Embryonal rhabdomyosarcoma & synovial sarcomas
- Adriamycin&Ifosphamide are the most active chemotherapeutic drugs

Correct Answer. d

(55). All of the following are features of bronchioalveolar carcinoma lung except:

a. Lepidic spread

b. Some variants carry better prognosis than adenocarcinomas

c. Mucinous variants are TTF-1 positive and CK-20 negative

d. Metastatic adenocarcinomas are TTF-1 negative and CK-20 positive

Solution. (c) Mucinous variants are TTF-1 positive and CK-20 negative
Ref:Read the text below
Sol:
- Lepidic spread (spread along surfactant) is the commonest mode of spread of bronchioalveolar carcinomas, but they have low
metastatic potential & lymphatic spread, therefore carry better prognosis as compared to adenocarcinomas
- Metastatic adenocarcinomas are differentiated from primary adenocarcinomas by 2 markers, i.e. TTF-1 (thyroid transcription factor-1)
& CK-20. Primary adenocarcinomas are TTF-1 positive & CK-20 negative, the only exception is mucinous variant of bronchioalveolar
carcinomas which behave like metastatic adenocarcinomas.

Correct Answer. c

(56). All of the following are true about carcinoid tumors except:

a. Appendix is the commonest site in GIT

b. Commonest site for malignant carcinoid is distal ileum

c. Elevated urinary 5-HIAA is diagnostic of carcinoid syndrome

d. Commonest valvular disease in carcinoid syndrome is pulmonary stenosis

Solution. (d) Commonest valvular disease in carcinoid syndrome is pulmonary stenosis


Ref:Read the text below
Sol:
o Bronchus is the commonest site for carcinoids & in the GIT, appendix is the commonest site
o Ileum is the commonest site for malignant carcinoids
o The commonest valvular lesion seen in carcinoids is tricuspid regurgitation

Correct Answer. d

Copyright © 2014 Delhi Academy of Medical Sciences, All Rights Reserved. 28/94
(57). Intratubular germ cell tumor is found adjacent to all except:

a. Spermatocytic seminoma

b. Dysgerminoma

c. Embryonal carcinoma

d. Choriocarcinoma

Solution. (a) Spermatocytic seminoma


Ref:Read the text below
Sol:
Intratubular germ cell neoplasia (ITGCN)
- ITGCN is seen adjacent to all germ cell tumors except for spermatocytic seminoma, epidermoid & dermoid cyst, pediatric teratoma&
yolk sac tumor
- ITGCN is the presumed precursor of all germ cell tumors
- It is encountered with high frequency in:
- Cryptorchidism
- Prior germ cell tumor
- Androgen insensitivity syndrome
- Gonadal dysgenesis syndrome
o Untreated ITGCN progresses to invasive germ cell tumor in approximately 50% of cases over 5 years of follow-up

Correct Answer. a

(58) Which of the following bone neoplasms is associated with a


. characteristic chromosomal translocation?

a. Ewing sarcoma

b. Osteosarcoma

c. Fibrous dysplasia

d. Eosinophilic granuloma

Solution. (a) Ewing sarcoma


Ref: Read the text below
Sol:

Correct Answer. a

Copyright © 2014 Delhi Academy of Medical Sciences, All Rights Reserved. 29/94
(59). Disease least likely to have a family predisposition

a. Germ cell cancer of testes

b. Breast cancer

c. Lung cancer

d. Ovarian cancer

Solution. (c) Lung cancer


Reference – Read the text below
Sol
- Roughly 100 syndromes of familial cancer have been reported, though many are rare.
- The majority are inherited as autosomal dominant traits, although some of those associated with DNA repair abnormalities
(xerodermapigmentosum, Fanconi anemia, ataxia telangiectasia) are autosomal recessives.
- Most of the genes responsible for the dominantly inherited cancer syndromes are tumour suppressor genes.
- The hallmarks of a tumour suppressor gene are as follows: (1) the germline mutation that affects one allele generally causes a loss of
function, (2) tumours also show loss of the second normal allele as a result of a somatic mutation, and (3) often the normal function of the
gene is to suppress unrestrained cellular growth or to promote differentiation.

Correct Answer. c

(60). Which one of the following drugs has the least bone marrow suppression?

a. Daunorubicin

b. Cisplatin

c. Etoposide

d. Vincristine

Solution. -NA-

Correct Answer. d

(61). Changes least likely with familial colonic cancer

a. APC gene mutation

b. DNA repair gene mutation

c. Chromosomal translocation

d. Microsatellite instability

Solution. (c) Chromosomal translocation


Reference – Read the text below
Sol
- Molecular studies have linked polyposis coli with a deletion on the long arm of Chr 5, including the APC gene – it is hypothesised that
the loss of this genetic material results in the absence of tumour suppressor genes whose presence would normally inhibit neoplastic
growth.
- HNPCC is linked to mutations caused by microsatellite instability in several mismatch repair genes, notably hMSH2&hMLH1.
- As a result there is defect repair of DNA mismatches with abnormal cell growth &tumour development. The majority of these mutations
are inactivating insertions or deletions.

Correct Answer. c

Copyright © 2014 Delhi Academy of Medical Sciences, All Rights Reserved. 30/94
(62). Tumor containing cells of all three germ layers is called

a. Leiomyoma

b. Squamous cell carcinoma

c. Adenocarcinoma

d. Teratoma

Solution. (d) Teratoma


Ref.:Robbin’s - 262
Sol :
Germ layers and tumor
- Great majority of the neoplasm are composed of cells representative of a single germ layer.
- Teratomas, in contrast, are made up of a variety of parenchymal cell types representative of more than one germ layer, usually all three.
- They arise from totipotent cells and so are principally encountered in the gonads.
- These totipotent cells differentiate along various germ lines, that can be identified as skin, fat, muscle, tooth structure, hair or any other
tissue of the body.
- Example is dermoid cyst of ovary is a benignteratoma (note – mature teratoma is benign, while immature teratoma is malignant.

Correct Answer. d

(63). Most common type of Hodgkin’s disease

a. Nodular sclerosis

b. Mixed cellularity

c. Lymphocyte predominant

d. Lymphocyte rich

Solution. (a) Nodular sclerosis


Ref.:Read the text below
Sol :

Correct Answer. a

Copyright © 2014 Delhi Academy of Medical Sciences, All Rights Reserved. 31/94
(64)
Gold standard test for diagnosis of PNH is
.

a. Coomb’s test

b. HAM test

c. Sucrose lysis test

d. Flow cytometry

Solution. (d) Flow cytometry


Ref.:Read the text below
Sol :

Correct Answer. d

(65). Which of the following is not a neoplasm epithelial origin?

a. Carcinoma of the esophagus

b. Carcinoma of the stomach

c. Rhabdomyisarcoma

d. Based cell Carcinoma

Solution. (c) Rhabdomyisarcoma


Ref: Read the text below
Sol:
- The terms”carcinoma” and “papilloma” are used to denote neoplasms that are epithelial origin.
- Rhabdomyosarcoma is a tumor as designated by sarcoma (of mesodermal origin) and the prefix rhabdo (of striated muscle origin).

Correct Answer. c

Copyright © 2014 Delhi Academy of Medical Sciences, All Rights Reserved. 32/94
(66). A biopsy of the liver shows tumor nodules composed of mucus-secreting adeno carcinoma.The most likely site of origin outside the liver is

a. Cerebellum

b. Prostate

c. Bladder

d. Colon

Solution. (d) Colon


Ref: Read the text below
Sol:
- This is a practical problem often faced by pathologists and clinicians when patients have symphatoms of metastasis before the primary
site is discovered.
- All primary tumors, with the exception of those of the cerebellum, can produce metastasis to the liver, but the colon is by far the most
common site and often produces a mucus-secreting adenocarcinoma arising from the glandular epithelium.

Correct Answer. d

(67). Which of the following characteristics is not typical of a sarcoma?

a. Pleomorphism

b. Blood vessel invasion

c. Lymphatic invasion

d. Mesodermal origin

Solution. (c) Lymphatic invasion


Ref: Read the text below
Sol:
- Although sarcomas may occasionally spread via the lymphatic invasion route, with very few exceptions they tend either to remain
locally invasive or to spread by the bloodstream.
- This probably is related to the fact that they are of mesodermal original and often have large vascular spaces that readily connect with
the vasculature of normal surrounding tissues, hence, their ability to spread more rapidly by that route.
- Pleomorphism or a variation in size and shape and appearance of the nuclei and the cell in general is relatively characteristic of these
neoplasms.

Correct Answer. c

(68). The process described as angioneogenesis is

a. Invasion of blood vessels by tumor cells

b. Blood-borne spread of tumor cells

c. Tumor arising from primitive blood-forming tissue

d. Growth of new blood vessels into a growing tumor

Solution. (d) Growth of new blood vessels into a growing tumor


Ref: Read the text below
Sol:
- As the name implies,angioneogenesis is the formation of new blood vessels.
- It has been shown that these often are of host origin, growing into the tumor and stimulated by the release from the tumor of
substances designated angiogenesis factors.

Correct Answer. d

Copyright © 2014 Delhi Academy of Medical Sciences, All Rights Reserved. 33/94
(69). Most common primary soft tissue tumor of chest wall

a. Solitary neurofibroma

b. Chondrosarcoma

c. Ewing’s sarcoma

d. Fibrosarcoma

Solution. (d) Fibrosarcoma .


Reference – Read the text below
Sol
- Fibrosarcoma is most common primary soft tissue tumor of chest wall.

Correct Answer. d

(70). Which one of the following marker is not positive for mesothelioma

a. Calretinin

b. WT – 1

c. Podoplanin

d. CD 68

Solution. (d) CD 68.


Reference – Read the text below
Sol
- CD68 is a marker is Histiocytes.
- It is not positive in mesothelioma.
- Mesothelioma shows positivity for Calretinin, WT – 1 and podoplakin.

Correct Answer. d

(71). The following are colonoscopic and histological images in a patient with bleeding per rectum. The likely diagnosis is?

a. Tubular adenoma

b. Villous adenoma

c. Adenocarcinoma

d. Carcinoid

Solution. (a) Tubular adenoma


Ref: Read the text below.
Sol:
- Colonoscopy is showing a pedunculated polyp and pedicle is seen on histology as well.
- Villous adenomas are sessile and tubulovillous adenomas share both features of villous and tubular adenomas

Correct Answer. a

Copyright © 2014 Delhi Academy of Medical Sciences, All Rights Reserved. 34/94
(72). A 68-year-old woman with a history of heavy tobacco use is found to have a solitary lung nodule on chest computed tomography.
Pathology from a recent bronchoscopy reveals adenocarcinoma. What further staging work-up is necessary for this patient before
surgical resection?

a. Brain magnetic resonance imaging

b. Positron emission tomography (PET) scan

c. Mediastinoscopy

d. Biopsy

Solution. (b) Positron emission tomography (PET) scan


Ref: Read the text below.
Sol:
- FDG([18F]2-Fluoro-2-deoxy-glucose)-PET for baseline locoregional staging is now part of the standard work-up for non-small cell lung
cancer (NSCLC).
- The true strength of this test for the mediastinum is its negative predictive value (NPV) when FDG uptake in the primary tumor is
sufficient and a central tumor or important hilar lymph node disease is absent.
- Two well-designed, prospective studies confirmed the high NPV (98%) of the mediastinum of patients with resectable NSCLC and that
significantly fewer invasive tests (ie, mediastinoscopy) were required in the PET group than in the traditional work-up group.

Correct Answer. b

(73). A 55-year-old woman recently completed induction chemotherapy for acute myelogenous leukemia (AML) with evidence of complete
remission on bone marrow biopsy. She subsequently was found to have a platelet count of 25,000/microliter with no clinical evidence of
bleeding.Platelet transfusion is necessary at what time:

a. Immediately

b. Within 24 hrs

c. Not required

d. Within 6 hrs.

Solution. (c) Not required


Ref: Read the text below.
Sol:
- Bleeding complications occur more frequently as the severity of thrombocytopenia increases, but only after the platelet count crosses a
threshold of about 10 to 30 x 103.
- A normal platelet count is not required to support hemostasis.
- Clinically important spontaneous bleeding does not occur unless the platelet count is very low or other disorders are present.
- In a study conducted at the National Cancer Institute to assess the risk for bleeding in patients with thrombocytopenia, the
investigators were not able to determine a threshold below which platelets should have prophylactic transfusion.
- However, following this study prophylactic transfusion became common practice for patients with a platelet count below 20 x 103.
Several subsequent randomized studies showed that using a platelet count < 10 x 103 as the trigger for prophylactic transfusion did not
increase the risk for bleeding.

Correct Answer. c

Copyright © 2014 Delhi Academy of Medical Sciences, All Rights Reserved. 35/94
(74). Obesity is a risk factor for which of the following types of cancer?

a. Renal

b. Endometrial

c. Esophageal adenocarcinoma

d. All of the above

Solution. (d) All of the above


Ref: Read the text below.
Sol:
- Excess body weight is becoming increasingly recognized as an important risk factor for cancer.
- Renehan and colleagues conducted a systematic review and meta-analysis of 141 articles that included 282,137 cases of cancer.
- Using data from these studies, they calculated the effect of gaining weight that was equivalent to an increase of 5 kg/m2 in body mass
index (BMI).
- Such a weight gain doubles the risk for esophageal adenocarcinoma in both sexes (relative risk [RR], 1.52; P < .0001). In women, it also
slightly more than doubles the risk for endometrial cancer (RR, 1.59; P < .0001) and increases risk for renal cancer by about a third (RR,
1.34; P < .0001).

Correct Answer. d

(75). Which of the following is NOT among the uses of PET imaging in the management of cervical cancer?

a. Initial diagnosis

b. Staging

c. Treatment planning

d. Assessment of prognosis

Solution. (a) Initial diagnosis


Ref: Read the text below.
Sol:
- The use of functional imaging techniques, such as FDG-PET and PET-CT, to manage patients with cervical cancer is constantly
increasing.
- Current roles include pretreatment staging and diagnosis of recurrent disease. Reports have also shown ability to predict survival based
on pre- and posttherapy scans.
- Cervical cancer spreads primarily through direct extension, although lymphatic and hematogenous spread can result in distant
metastases.
- Traditionally, clinical examination and cross-projectional imaging have been used to stage cervical cancer but are accurate in only 60%
of cases.
- Detection of lymphatic metastases seems to be more successful with functional imaging techniques, such as FDG-PET or PET-CT.

Correct Answer. a

(76). Which of the following staging system is used for malignant pleural mesothelioma

a. Lewis staging system

b. Herper staging system

c. RAI staging system

d. Brigham staging system

Solution. (d) Brigham staging system .


Reference – Read the text below
Sol
- Brigham staging system is usually used for staging of malignant pleural mesothelioma.

Correct Answer. d

Copyright © 2014 Delhi Academy of Medical Sciences, All Rights Reserved. 36/94
(77). Unfavorable prognostic factors for malignant pleural mesothelioma include all of the following except

a. Sarcomatoid type

b. Male gender

c. Age < 20 years

d. Aneuploidy

Solution. (c) Age < 20 years.


Reference – Read the text below
Sol
- Advance age is a bad prognostic factor for malignant mesothelioma.
- Age < 20 yrs is a good prognostic factor.

Correct Answer. c

(78). PTAH (Phosphotungstic Acid hemotoxylin )stain is useful for diagnosis of

a. Alveolar soft part sarcoma

b. Rhabdonyo sarcoma

c. Fibrosarcoma

d. All of them

Solution. (b) Rhabdonyo sarcoma


Reference – Read the text below
Sol
- PTAH stain is used to diagnose Rhabdomyosarcoma

Correct Answer. b

(79). The commonest side effect of 5-fluorouracil is

a. Bone marrow depression

b. Gastrointestinal tract ulceration

c. Hypersensitivity reaction

d. Acute chest pin.

Solution. (b) Gastrointestinal tract ulceration


Reference – Read the text below
Sol
- During 5-FU therapy, the earliest untoward symptoms are anorexia and nausea. These are followed by stomatitis and diarrhea.
- Mucosal ulcerations occur throughout the g.i.t. and may lead to fulminant diarrhea, shock and death, specially with 5-FU continuous
infusions.
- However, the bolus-dose regimen causes bone marrow depression as it major side effect.

Correct Answer. b

Copyright © 2014 Delhi Academy of Medical Sciences, All Rights Reserved. 37/94
(80). The drug of choice in multiple myeloma is

a. Prednisolone

b. Cyclophosphamide

c. Melphalan

d. Mechlorethamine

Solution. (c) Melphalan


Reference – Read the text below
Sol
- Melphalan hydrochloride (trade name Alkeran) is a chemotherapy drug belonging to the class of nitrogen mustard alkylating agents
- It is used to treat multiple myeloma and ovarian cancer, and occasionally malignant melanoma.

Correct Answer. c

(81). Which of the following is not an antimetabolite

a. Methotrexate

b. Pentamidine

c. Pyrimethamine

d. Cytosine arabinoside.

Solution. (b) Pentamidine


Reference – Read the text below
Sol
- Pentamidine is used in Kala-azar and pneumocystis pneumonia, as a second line drug. It acts by multiple mechanisms, precisely not
known.
- Mainly it targets DNA kinetoplast and topoisomerase II.
The antimetabolites used as anticancer drugs are:
(i) Folate antagonist : Methotrexate
(ii) Purine antagonists: 6-Marcaptopurine, 6-thioguanine, azathioprine.
(iii) Pyrimidine antagonists: 5-fluorouracil, cytosine arabinoside (cytarabine).
- Pyrimethamine is also folate antagonist like methotrexate.
- But pyrimethamine is highly selective for plasmodial DHF Rase while methotrexate is selective for mammalian DHF Rase (trimethoprim
is highly selective for bacterial DHF Rase).
- Hence, the difference in their actions despite of the same mechanism.

Correct Answer. b

(82). Which of the following is a common side effect of cisplatin

a. Diarrhea

b. Nausea

c. Pulmonary fibrosis

d. Alopecia.

Solution. (b) Nausea


Reference – Read the text below
Sol
- Nausea is the commnest side effect of cisplatin, as it is probably the most emetogenic drug.

Correct Answer. b

Copyright © 2014 Delhi Academy of Medical Sciences, All Rights Reserved. 38/94
(83). Which of the following tumors is least associated with hormone dependence?

a. Carcinoma of the prostate

b. Carcinoma of the breast in females

c. Carcinoma of the thyroid

d. Carcinoma of the colon

Solution. (c) Carcinoma of the thyroid


Ref: Read the text below
Sol:
- There is little direct evidence to suggest that colonic carcinoma is directly under hormonal influence.
- All of the others may be altered by treatment or manipulation, such as ablation of estrogen producing tissues in carcinoma of the breast
or castration and administration of estrogen in carcinoma of the prostate.
- In the thyroid, the cancer actually may produce thyroid hormone and be destroyed by uptake of radiolabeled iodine by the functioning
tumor cells.
- Not all of the cancer cells in prostate, breast, or thyroid, however , are hormone dependent.
- This depends on the presence of estrogen or progesterone receptors in breast cancer, the production of hormone receptors in prostate
cancer, and the actual production of a functioning hormone in carcinoma of the thyroid.

Correct Answer. c

(84). Which of the following does not act by IL-2 modification

a. MMF

b. Cyclosporine

c. Tacrolimus

d. Rapamycin

Solution. (a) MMF


Reference – Read the text below
Sol
- Mycophenolate mofetil (MMF) is a immunosupressant and prodrug of mycophenolic acid, used extensively in transplant medicine. Its
mode of action is as a reversible inhibitor of Inositol monophosphate dehydrogenase (IMPDH) in purine biosynthesis. MMF is selective
for the de novo pathway critical to lymphocytic proliferation and activation. Other cells are able to recover purines via a separate,
scavenger, pathway and are thus able to escape the effect.
- Tacrolimus (also FK-506 or Fujimycin) is an immunosuppressive drug whose main use is after allogeneic organ transplant to reduce the
activity of the patient's immune system and so lower the risk of organ rejection. It reduces T-cell and interleukin-2 (IL-2) activity.

Correct Answer. a

Copyright © 2014 Delhi Academy of Medical Sciences, All Rights Reserved. 39/94
(85). Which one of the Toll - like receptors respond to virus derived molecules?

a. TLR2

b. TLR3

c. TLR4

d. TLR5

Solution. (b) TLR3


Reference – Read the text below
Sol

Correct Answer. b

Copyright © 2014 Delhi Academy of Medical Sciences, All Rights Reserved. 40/94
(86). All are prognostic markers of ALL except

a. Age

b. Sex

c. Hyperdiploidy

d. Neurocutaneous markers

Solution. (d) Neurocutaneous markers


Reference – Read the text below
Sol
The survival rate has improved from zero four decades ago, to 20-75 percent currently,
largely due to clinical trials and improvements in bone marrow transplantation (BMT) and stem cell transplantation (SCT) technology.
Five-year survival rates evaluate older, not current, treatments. New drugs, and matching
treatment to the genetic characteristics of the blast cells, may improve those rates. The prognosis
for ALL differs between individuals depending on a variety of factors:
- Sex: females tend to fare better than males.
- Ethnicity: Caucasians are more likely to develop acute leukemia than African- Americans, Asians and Hispanics and tend to have a
better prognosis than non-Caucasians.
- Age at diagnosis: children between 1–10 years of age are most likely to develop ALL and to be cured of it. Cases in older patients are
more likely to result from chromosomal abnormalities (e.g. the Philadelphia chromosome) that make treatment more difficult and
prognoses poorer.
- White blood cell count at diagnosis of less than 50,000/μl
- Cancer spread into the Central_nervous_system (brain or spinal cord) has worse outcomes.
- Morphological, immunological, and genetic subtypes
- Patient's response to initial treatmenT
- Genetic disorders such as Down's Syndrome

Correct Answer. d

Copyright © 2014 Delhi Academy of Medical Sciences, All Rights Reserved. 41/94
(87)
Which of the following liver tumors occur in infancy?
.

a. Fibrolamellar carcinoma

b. Hepatoblastoma

c. Epitheloid hemangioendothelioma

d. All of the above

Solution. (b) Hepatoblastoma


Reference – Read the text below
Sol

Correct Answer. b

Copyright © 2014 Delhi Academy of Medical Sciences, All Rights Reserved. 42/94
(88). “Dry-tap” in bonemarrow aspirate is characteristic of which variant of AML:

a. M4

b. M5

c. M6

d. M7

Solution. (d) M7.


Reference – Read the text below
Sol
- “Dry-tap” in bone marrow aspirate is characteristic of M7 variant of AML.
- The ‘Dry – tap” is due to prominent bone marrow fibrosis.

Correct Answer. d

(89). Which of the following statement about PTLD (post-transplant lympho proliferative disorder) is incorrect

a. Stronger the immunosuppressive regimen, such as use of anti-OKT3, the heigher the risk

b. Often appear several months to year after the transplant, with a median interval of 4.5 – 11 months

c. PTLDs occurring late after solid organ transplantation are less frequently associated with EBV (44%)

d. Hodqkin lymphoma like PTLD appear after 17 to 19 months of transplant

Solution. (d) Hodqkin lymphoma like PTLD appear after 17 to 19 months of transplant.
Reference – Read the text below
Sol
- Hodgkin lymphoma like PTLD usually appear after 49 months of transplant.
- It is a B cell lympho – proliferative malignancy.

Correct Answer. d

(90). “Compartmentalization phenomenon” is characteristic of

a. Sezary syndrome

b. T-cell granular lymphocytic leukemia

c. PTCL

d. None of them

Solution. (c) PTCL .


Reference – Read the text below
Sol
- “Compartmentalization phenomenon” is characteristic of PTCL; caused due to thin bands of collagen traversing through the lesion.

Correct Answer. c

Copyright © 2014 Delhi Academy of Medical Sciences, All Rights Reserved. 43/94
(91). H-RS like cells is characteristic of which infection

a. Hepatitis B

b. Hepatitis C

c. Hepatitis E

d. EBV virus

Solution. (d) EBV virus.


Reference – Read the text below
Sol
- H-RS like cells is characteristic of Infectious mononucleosis infection caused by EBV virus.

Correct Answer. d

(92). Highest concentration of Rs-cell is seen in which variant of hodqkin lymphoma

a. NLPHD

b. Nodular sclerosis

c. Mixed cellularity

d. Lymphocyte depleted

Solution. (c) Mixed cellularity.


Reference – Read the text below
Sol
- Mixed cellularity variant of Hodgkins lymphoma shows highest concentration of RS cell.

Correct Answer. c

Copyright © 2014 Delhi Academy of Medical Sciences, All Rights Reserved. 44/94
(93). Cd38 is a poor prognostic marker of which of the following

a. CML

b. AML

c. CLL

d. ALL

Solution. (c) CLL.


Reference – Read the text below
Sol
- CLL CD 38 poses poor prognosis for CLL.

Correct Answer. c

(94). Which of the following bears a bad prognostic factor for ALL

a. Hyperploidy

b. Hypoploidy

c. Both

d. None of them

Solution. (b) Hypoploidy


Reference – Read the text below
Sol
- Hypoploidy is a bad prognostic factor for ALL.
- Hyperploidy, on the other band, bears a good prognosis for ALL patients.

Correct Answer. b

(95). “Sarcoma botryoides” is a variant of

a. Liposarcoma

b. Rhabdomyosarcoma

c. Synovial sarcoma

d. None of them

Solution. (b) Rhabdomyosarcoma.


Reference – Read the text below
Sol
- Sarcoma botryoides or botryoid sarcoma or botryoid rhabdomyosarcoma is a subtype of embryonal rhabdomyosarcoma, that can be
observed in the walls of hollow, mucosa lined structures such as the nasopharynx, common bile duct, urinary bladder of infants and
young children or the vagina in females, typically younger than age 8.
- The name comes from the gross appearance of "grape bunches"
- It shows positivity for Desmin, Myogenin and Myo-D.

Correct Answer. b

Copyright © 2014 Delhi Academy of Medical Sciences, All Rights Reserved. 45/94
(96). One of the following cyclins is associated with breast cancer:

a. Cyclin A

b. Cyclin B

c. Cyclin D

d. Cyclin E

Solution. (d) Cyclin E


Reference – Read the text below
Sol
- Cyclin E and its low molecular weight forms are over expressed in breast cancers, and the level of expression correlates with disease
progression and survival.

Correct Answer. d

(97). Carcinoid tumors of the lung originate from which of the following cell types:

a. Ciliated cell

b. Clara cell

c. Kulchitsky (K) cell

d. Mucus (goblet) cell

Solution. (c) Kulchitsky (K) cell


Reference – Read the text below
Sol
- Gastrointestinal carcinoid, also called carcinoid tumor, is the most common primary tumor of the small bowel and appendix.
Gastrointestinal carcinoid accounts for more than 95% of all carcinoids.
- The tumors arise from enterochromaffin cells of Kulchitsky, which are considered neural crest cells situated at the base of the crypts of
Lieberkühn. Gastrointestinal carcinoids account for 1.5% of all gastrointestinal tumors.
- The tumors elaborate serotonin and other histaminelike substances that normally are transported to the liver, where they are
metabolized.
- Most tumors are clinically silent, but they may cause pain or intestinal obstruction, weight loss, a palpable mass, or, rarely, bowel
perforation.
- Carcinoid syndrome occurs when the humoral load exceeds the capacity of monoamine oxidase (MAO) present in the liver and lung to
metabolize serotonin.
- Most patients with carcinoid syndrome have liver metastases from a bowel carcinoid, although in rare cases, the humoral load from a
primary tumor may overwhelm the liver and the capacity of the lungs to metabolize serotonin.
- Rarer still is carcinoid syndrome that develops in patients with noncarcinoid malignant tumors and dermatomyositis.

Correct Answer. c

Copyright © 2014 Delhi Academy of Medical Sciences, All Rights Reserved. 46/94
(98). All are differential of ALL except

a. Aplastic anemia

b. Myelodysplasia

c. Neuroblastoma

d. Hypereosinophilic syndrome

Solution. (d) Hypereosinophilic syndrome


Reference – Read the text below
Sol
Differential diagnosis
- Infections e.g. EBV, Parvovirus B19
- Myeloproliferative or lymphoproliferative disorders
- Myelodysplasia
- Aplastic anaemia
- ITP
- Lymphoma
- Metastatic cancer
- Other cancers of childhood e.g. neuroblastoma, rhabdomyosarcoma
- Osteomyelitis
- Juvenile chronic arthritis

Correct Answer. d

(99). Necrotising Hystiocytic Lymphadinitis on histopathology is seen in:

a. Castelmans Disease

b. Kikuchi’s Disease

c. Kimura Disease

d. Hodgkins Disease

Solution. (c) Kikuchi’s Disease


Reference – Read the text below
Sol
- It is a differential diagnosis of tubercular lymphadenitis

Correct Answer. c

(100). Which of the following tumors is not commonly known to increase in pregnancy?

a. Glioma

b. Angiomas

c. Meningioma

d. Neurofibroma.

Solution. (a) Glioma


Reference – Read the text below
Sol
“Neurofibromas and Meningiomas although brain tumors are not specifically related to gestation but meningiomas, angiomas and
neurofibromas are though to grow more rapidly with pregnancy.”

Correct Answer. a

Copyright © 2014 Delhi Academy of Medical Sciences, All Rights Reserved. 47/94
(101). All of the following are biochemical markers included for triple test except:

a. Alfa feto protein [AFP]

b. Human chorionic gonadotropin [HCG]

c. Human placental lactogen [HPL]

d. Unconjugated oestriol.

Solution. (c) Human placental lactogen [HPL]


Reference – Read the text below
Sol
- Triple test is a combined biochemical test.
- It includes the estimation of: Maternal serum alpha feto protein (MSAFP) Human chorionic gonadotropin (hCG) Unconjugated oestriol
(UE3).
- Is performed at 16-18 weeks. Is used for detection of Down’s syndrome. In an affected pregnancy, level of MSAFP and UE3 tend to be
low while that of HCG is high.
- Triple test is used as a screening test. (For confirmation of Down’s syndrome aminocentesis has to be done.)

Correct Answer. c

(102). In Burkitt’s lymphoma, which one of the following mechanisms is responsible for the activation of the c-myc gene?

a. Point mutation

b. Gene amplification

c. Chromosomal translocation

d. Gene deletion

Solution. (c) Chromosomal translocation


Reference – Read the text below
Sol
In Burkitt’slymphome, there is one of three translocations involving chromosome 8q24, where the c-myc gene has been mapped. Burkitt’s
lymphoma
- Demonstration of a very high proliferative fraction and the presence of the t(8;14) or one of its variants, t(2;8) or t(8;22) (c-mycand the
a light chain gene), can be confirmatory.
- Burkitt’s cell leukemia is recognized by the typical monotonous mass of medium-sized cells with round nuclei, multiple nucleoli, and
basophilic cytoplasm with cytoplasmic vacuoles.

Correct Answer. c

(103). A keloid is

a. A protruding tumor like scar

b. The outcome of most infected wounds

c. A form of early granulation tissue

d. Benign tumor of melanocytic origin

Solution. (a) A protruding tumor like scar


Ref: Read the text below
Sol:
- Keloid is an overxuberantproduction of collagen bundles, usually following a wound that may be trivial.
- It often is a tumor like mass but is not neoplastic in the true sense of the world in that the phenomenon is self- limiting and does not
progress or spread or metastasize as does not progress or spread or metastasize as do other forms of neoplastic growth.
- The formation of keloid often is a racially determined condition, andblacks have a high propensity for producing such scarring, with
gross distortion, particularly in response to burns.

Correct Answer. a

Copyright © 2014 Delhi Academy of Medical Sciences, All Rights Reserved. 48/94
(104). A tumor composed of tissues representing all three embryologic germ layers is called

a. Adenocarcinoma

b. Carcinosarcoma

c. Mixed mesodermal tumor

d. Teratoma

Solution. (d) Teratoma


Ref: Read the text below
Sol:
- Teratoma, which may be benign or malignant, is the only tumor type in which all three germ cell layers are represented.
- All the others contain one or two cell types.
- Teratomas typically are seen in the ovaries or testis but also occur in other sites , such as the mediastinum and retroperitoneum.

Correct Answer. d

(105). A biopsy shows what appears to be an abnormal amount and arrangement of normal tissue that is appropriate or normal for the area in
which the tissue arises. This is best described as

a. Teratoma

b. Mixed tumor

c. Hamartoma

d. Embryonal tumor

Solution. (c) Hamartoma


Ref: Read the text below
Sol:
- Hamartoma is a conglomeration of tissues that are normal to the area but haphazardly arranged in an abnormal fashion.
- All of the neoplasis listed contain a mixture of different cell types, but only in the hamartoma are the cells normal to that particular
area.
- These are best regarded as developmental anomalies rather than true neoplasms, and they are never malignant and, therefore, do not
metastasize.

Correct Answer. c

(106). Which of the following best describes the phenomenon of epithelial dysplasia?

a. An increase in thickness of the epithelium because of increased number of cells

b. A decrease in thickness of the epithelium owing to decrease in number of dividing cells

c. An irregular proliferation and maturation of cells throughout the layers of the epithelium

d. An increase in thickness of the epithelium owing to enlargement of the component cells

Solution. (c) An irregular proliferation and maturation of cells throughout the layers of the epithelium
Ref: Read the text below
Sol:
- As described , dysplasia is an abnormal sequence of changes in maturation or development of cell types in a tissue or organ
- It is best seen in the cervix of the uterus, where normally an ordered regular maturation of basal cells results in flattened squamous
epithelium at the surface.
- In dysplasia, nucleated cells appear throughout the layers even at the surface and can be shared and thus recognized in cytologic
preparations.
- Dysplasia, in company with dyscrasia and dystrophy, is often used in describing tissues with other perversions of cell growth. All are
self –limiting and distinct from neoplasia.

Correct Answer. c

Copyright © 2014 Delhi Academy of Medical Sciences, All Rights Reserved. 49/94
(107). Which of the following is the least correct statement?

a. The incidence of malignancy tends to increase with age

b. Hormones may inhibit or promote tumor growth

c. Certain tumors characteristically appear in certain age groups

d. Carcinoma of the breast is more frequent in females than in males

Solution. (b) Hormones may inhibit or promote tumor growth


Ref: Read the text below
Sol:
- It is true that incidence of malignancy tends to increase with age. It also is true that hormones may inhibit or promote tumor growth.
- Certain tumors characteristically appear in certain age groups.
- Carcinoma of the breast, however, is more frequent in the female than the male, and cancer of the male breast , although it does occur,
is particularly rare.

Correct Answer. b

(108). PMS2 gene association is which of the following cancers :

a. Ataxia telengiectasia

b. HNPCC

c. Bloom’s syndrome

d. Fanconi’s anemia

Solution. (b) HNPCC


Ref:Read the text below
Sol :
- PMS2, MLH-1, MSH-2 & MSH-6 are associated with Microsatteliteinstability(MSI) in HNPCC.
- Microsattelites are random repeats of 1-6 nucleotides that are scattered throughout the genome, any lengthening or shortening is
defined as MSI

Correct Answer. b

(109). Amplification of CDK4 is seen in all except:

a. Leiomyosarcomas (LMS)

b. Alveolar rhabdomyosarcomas (ARMS)

c. Embryonal rhabdomyosarcomas

d. Carcinomas

Solution. (d) Carcinomas


Ref:Read the text below
Sol :
- We evaluated amplification and overrepresentation of CDK4, MDM2, GLI and SAS genes of the 12q13-15 region, in a group of soft
tissue sarcomas including leiomyosarcomas (LMS), alveolar rhabdomyosarcomas (ARMS) and embryonal (anaplastic and classic variants)
rhabdomyosarcomas (ERMS),

Correct Answer. d

Copyright © 2014 Delhi Academy of Medical Sciences, All Rights Reserved. 50/94
(110). Cell cycle inhibitor having no independent role of its own?

a. P21

b. P16

c. P27

d. P57

Solution. (d) P57


Ref:Read the text below
Sol :
- Cell cycle inhibitors are broadly divided into 2 families- Cip/kip family and INK4a family.
- Cip/kip family has p21 (CDKN1A), p27 (CDKN1B), p57 (CDKN1C).
- INK4a family has p16 (CDKN2A), p15 (CDKN2B), p18 (CDKN2C), p19 (CDKN2D). INK4a family of cell cycle inhibitors has a selective
effect on cyclinD/CDK4 and cyclinD/CDK6 whereas cip/kip family is present throughout the cell cycle

Correct Answer. d

(111). ATM gene mediates its action by all except:

a. Activation of CDC25 phosphatase

b. As a part of BRCA-1 & BRCA-2 network

c. Disruption of cyclin B/CDK1 complex

d. Through p53

Solution. (a) Activation of CDC25 phosphatase


Ref:Read the text below
Sol :
- ATM (Ataxia- telengiectasia mutated) is a checkpoint component (in addition to p53) that is activated by mechanisms that sense double-
stranded DNA breaks.
- Transmits signals to arrest the cell cycle after DNA damage. Acts through p53 in the G1S checkpoint.
- At the G2M checkpoint, it acts through both p53-dependent mechanisms and through inactivation of CDC25 phosphatase, which
disrupts the cyclinB-CDK1 complex.
- It is a component of a network of genes that include BRCA1 & BRCA2 which link DNA damage with cell cycle arrest and apoptosis.

Correct Answer. a

(112). Rb plays a role in cell cycle by action on all except:

a. G2-M transition

b. Chromatin remodelers

c. E2F

d. None of above

Solution. (a) G2-M transition


Ref:Read the text below
Sol :
- Early in G1, RB is in its hypophosphorylated active form, and it binds to and inhibits the E2F family of transcription factors, preventing
transcription of cyclin E.
- Hypophosphorylated RB blocks E2F-mediated transcription in atleast 2 ways. First, it sequesters E2F, preventing it from interacting
with other transcriptional activators.
- Second, RB recruits chromatin-remodeling proteins, such as hsitone deacetylases and histone methyltransferases, which bind to the
promoters of E2F responsive genes such S cyclin E.
- These enzymes modify chromatin so as to make promoters insensitive to transcription factors. Mitogenicsignaling leads to cyclin D
expression and activation of cyclin D-CDK4/6 complexes.
- These complexes phosphorylate RB, inactivating the protein and releasing E2F to induce target genes such as cyclin E.
- Expression of cyclin E then stimulates DNA replication and progression through cell cycle.

Correct Answer. a

Copyright © 2014 Delhi Academy of Medical Sciences, All Rights Reserved. 51/94
(113). Analysis of metastatic tumor cells has revealed expression of factors promoting tissue invasion.Which of the following is not among such
factors:

a. MMP-9

b. TPA

c. Cathepsins

d. Urokinase

Solution. (c) Cathepsins


Ref:Read the text below
Sol :
- Evidence for the involvement of tissue degradative enzymes in neoplastic invasion is convincing.
- The production and activation of specific tissue destructive enzymes such as lysosomal hydrolases and collagenases has been
documented in tumor invasion.
- Destruction of host tissue by hydrolytic enzymes, aided by pressure atrophy and occlusion of blood and lymph vessels by an expanding
tumor mass, facilitates infiltration of neoplastic tissue.
- Enzymes involved in invasion by tumor cells include collagenases, cathepsins, matrix metalloproteinases (MMP4 &MMP9) and
plasminogen activators.

Correct Answer. c

(114). The most common dose limiting side effect of planned chemotherapy regimens is which of the following:

a. Pulmonary fibrosis

b. Gastrointestinal ulceration

c. Hematologic suppression

d. Hepatotoxicity

Solution. (c) Hematologic suppression


Ref:Read the text below
Sol :
- In chemotherapeutic regimens, it is important not only to deliver adequate doses with each treatment cycle but also to deliver them in
an orderly fashion.
- The interval between each treatment cycle is determined by toxic effects experienced by normal tissues and the amount of time
required for resolution of effects.
- For most agents, the dose-limiting toxicity is myelosupression, usually leucopenia&/or thrombocytopenia. Nadir blood counts are
reached approximately 14 days after completion of each cycle and begin to improve 3 to 5 days later, often with complete resolution by
day 28.
- The resilience of bone marrow is dependent upon prior chemotherapy & radiotherapy. Agents such as busulphan, mitomycin C,
procarbazine and the nitrosoureas display delayed myelosupression, often upto 4 weeks after the initiation of therapy & lasting several
weeks.
- Therapy with these drugs may be delivered only every 6-8 weeks; in addition, therapy with these agents may lead to chronic cumulative
myelosupression&, in some cases, marrow failure.

Correct Answer. c

Copyright © 2014 Delhi Academy of Medical Sciences, All Rights Reserved. 52/94
(115). Resistance of tumors to multiple chemotherapeutic drugs is often due to the MDR (multiple drug resistance) gene. This gene encodes a
protein that acts by which of the following mechanisms?

a. As a transmembrane efflux pump for chemotherapeutic drugs

b. As a DNA repair molecule

c. As an isoform of superoxide dismutase

d. As a membrane stabilizer

Solution. (a) As a transmembrane efflux pump for chemotherapeutic drugs Ref:Read the text below
Sol :
- Resistance to chemotherapeutic drugs may become manifest during the administration of drugs of very different classes, with a variety
of presumed intracellular targets.
- Such resistance, a function separate from the state of proliferation of the malignant cells, has been termed multidrug resistance.
- This form of resistance is mediated by the cell surface protein, P-glycoprotein, a product of MDR-1 gene. A magnesium-dependent
ATPase, P-glycoprotein acts as a transmembrane efflux pump and appears to serve as a cellular detoxifier and possibly as a chloride
pump.
- P-glycoprotein causes the extrusion of diverse agents such as anthracyclines, epipodophyllotoxins, vinca alkaloids and actinomycins out
of the cell before they are able to reach their intracellular targets. The protein is normally found on luminal surface of such organs as the
colon and rectum, small intestine, proximal tubules of kidney and gravid uterus; the acinar and bile canalicular surfaces of pancreatic
and hepatic parenchymal cells, respectively.
- P-glycoprotein has been found in tumors derived from these organs.
- P-glycoprotein can be induced in vitro by exposing cell lines to increasing concentrations of various agents, a phenomenon which also
extends to tissues and cell lines not known to normally express significant amounts of P-glycoprotein, such as melanoma, ovarian
carcinoma, small cell carcinoma of lung, and adenocarcinoma of breast.

Correct Answer. a

(116). Telomerase activation is brought about by?

a. E6

b. E7

c. Both of above

d. None of above

Solution. (a) E6
Ref:Read the text below
Sol :
- The E7 protein of HPV binds to the RB protein (binding occurs in the same RB pocket that normally sequesters E2F transcription
factors) and displaces the E2F transcription factors, promoting progression through the cell cycle.
- Of note, E7 protein from high-risk HPV types has a higher affinity for RB than does E7 from low-risk types.
- E7 also inactivates the CDKIs p21 & p27. E7 proteins also bind and presumably activate cyclins E & A.
- The E6 protein has complementary effects.
- It binds to and mediates the degradation of p53 and BAX, a proapoptotic member of the BCL2 family, and it activates telomerase.

Correct Answer. a

Copyright © 2014 Delhi Academy of Medical Sciences, All Rights Reserved. 53/94
(117). Secondary myelofibrosis includes all of the following except:

a. CML

b. PV

c. CNL

d. ET

Solution. (c) CNL


Ref:Read the text below
Sol :
Diagnostic criteria for Primary myelofibrosis: Major criteria:
- Presence of megakaryocytic proliferation & atypia, usually accompanied by either reticulin / collagen fibrosis OR in the absence of
significant reticulin fibrosis, the magakaryocyte changes must be accompanied by an increased bone marrow cellularity characterized by
granulocytic proliferation & often decreased erythropoiesis
- Not meeting WHO criteria for polycythemia vera, BCR-ABL+ CML, MDS or other myeloid neoplasms
- Demonstration of JAK2 mutation
Minor criteria:
o Leucoerythtoblastosis
o Increased LDH
o Anaemia
o Splenomegaly
Diagnosis requires meeting all 3 major & 2 minor criteria.

Correct Answer. c

(118). WHO criteria for Essential thrombocythemia include all except:

a. Platelet count > 450,000

b. Megakaryocytic proliferation characterized by enlarged mature cells

c. JAK2 mutation

d. Platelet count > 600,000

Solution. (d) Platelet count > 600,000


Ref:Read the text below
Sol : Diagnostic criteria for Essential thrombocythemia:
o Sustained platelet count >450,000
o Bone marrow biopsy specimen showing proliferation mainly of megakaryocytic lineage with increased numbers of enlarged, mature
megakaryocytes. No significant increase or left-shift of neutrophil granulopoiesis or erythropoiesis
o Not meeting WHO criteria for polycythemia vera, primary myelofibrosis, BCR- ABL+ CML or MDS or other myeloid neoplasm
o Demonstration of JAK2 mutation
Diagnosis requires meeting all 4 criteria.

Correct Answer. d

Copyright © 2014 Delhi Academy of Medical Sciences, All Rights Reserved. 54/94
(119). True about acute erythroid leukemia is all of the following except:

Copyright © 2014 Delhi Academy of Medical Sciences, All Rights Reserved. 55/94
a. D/D for subtype 6a is PV

b. Expression of glycphorin A & hemoglobin A

Copyright © 2014 Delhi Academy of Medical Sciences, All Rights Reserved. 56/94
c. Acute megakaryoblastic leukemia most difficult D/D

Copyright © 2014 Delhi Academy of Medical Sciences, All Rights Reserved. 57/94
d. Erythroid blasts negative for glycophorin

Copyright © 2014 Delhi Academy of Medical Sciences, All Rights Reserved. 58/94
Solution. (d) Erythroid blasts negative for glycophorin
Ref:Read the text below
Sol :

Correct Answer. d

Copyright © 2014 Delhi Academy of Medical Sciences, All Rights Reserved. 59/94
(120). A 50 year old male pt. presents with generalized lymphadenopathy. Biopsy shows a nodular pattern with RS like cells. Cells express
CD20, CD45 and are negative for CD15 Diagnosis?

a. DLBCL

b. CLL

c. Burkitt’s lymphoma

d. NLPHL

Solution. (d) NLPHL


Ref:Read the text below
Sol :
NODULAR PREDOMINANT HODGKIN’S LYMPHOMA:
- 5% OF Hodgkin’s lymphoma
- Male, 30-50 yrs
- Usually involves cervical, axillary, inguinal ln’s
- LN architecture totally or partially replaced predominantly consisting of small lymphocytes, histiocytes, epithelioid histiocytes and
intermingled L&H CELLS
- Immunophenotype: CD45 POSITIVE, CD20 POSITIVE, CD15&CD30 NEGATIVE

Correct Answer. d

Copyright © 2014 Delhi Academy of Medical Sciences, All Rights Reserved. 60/94
(121). True about follicular lymphoma is:

a. Neoplasm of centroblasts

b. Grade 3a shows centrocytes

c. Cells are CD19, CD20 -ve

d. Cells are +ve for ALK-1

Solution. (b) Grade 3a shows centrocytes


Ref:Read the text below
Sol :

Correct Answer. b

Copyright © 2014 Delhi Academy of Medical Sciences, All Rights Reserved. 61/94
(122). A 63-year-old man with a remote history of tobacco use presents with hematuria. He undergoes a transurethral resection (TUR), which
reveals T1 superficial bladder cancer.What is his risk for bladder cancer recurrence in the next 5 years without further therapy?

a. 10%

b. 30%

c. 50%

d. 70%

Solution. (d) 70%


Ref: Read the text below.
Sol:
- TUR remains the surgical mainstay for the diagnosis and treatment of stages Ta and T1 bladder cancer.
- After this procedure, the 10-year disease-specific survival for Ta tumors is 85% and for T1 tumors it is 70%.
- The initial TUR provides pathologic material to determine the histologic type, grade, and depth of invasion.
- The findings help to direct additional therapy, dictate the follow-up schedule, and indicate prognosis.
- In a literature review of more than 600 cases of T1 bladder tumors treated only by TUR, 75% to 90% recurred by 5- and 10-year follow-
up.
- A third of the cancers progressed to muscle invasion within 5 years, 39% had progressed by 10 years, and 53% by 15 years.

Correct Answer. d

(123). A 35-year-old woman presents with cervical lymphadenopathy and symptoms of fever, night sweats, and weight loss. An excisional biopsy
of a supraclavicular lymph node reveals nodular sclerosing Hodgkin's lymphoma. Staging PET-CT shows bulky mediastinal
lymphadenopathy (mediastinal/thoracic ratio > 1/3), and she has a baseline erythrocyte sedimentation rate (ESR) of 50 mm/h.What
favorable prognostic feature does this patient have for early-stage Hodgkin's lymphoma?

a. Age < 50 years

b. Presence of B symptoms

c. ESR > 30 mm/h

d. All of the above

Solution. (a) Age < 50 years


Ref: Read the text below.
Sol:
The European Organization for Research and Treatment of Cancer (EORTC) defined prognostic features for favorable and unfavorable
stage I or II Hodgkin's lymphoma based on age, number of involved nodal areas, ESR, bulky disease, and B symptoms. Favorable features
include age
- < 50
- No B symptoms or elevated ESR,
- ≤ 3 involved lymph node regions
- No bulky disease.

Correct Answer. a

Copyright © 2014 Delhi Academy of Medical Sciences, All Rights Reserved. 62/94
(124). Temozolomide has emerged as an agent that is active against glioblastoma multiforme (GBM). What is the mechanism of action of this
chemotherapeutic drug?

a. Anthracycline

b. Antimetabolite

c. Alkylating agent

d. Antitubulin agent

Solution. (c) Alkylating agent


Ref: Read the text below.
Sol:
- Temozolomide (TMZ) is an orally administered alkylating agent of the imidazotetrazine class that is chemically related to dacarbazine.
- It is rapidly and completely absorbed and passes through the blood-brain barrier, creating cerebrospinal fluid concentrations that are
20% to 40% of plasma levels.
- The DNA repair protein MGMT removes alkyl groups from the O6 position of guanine, a target of alkylating agents, such as TMZ.
- In tumors, high expression of MGMT has been associated with resistance to treatment with alkylating agents.
- Inactivation of the MGMT gene by promoter methylation diminishes DNA repair activity and has been associated with longer overall
survival in patients with GBM who are treated with alkylating agents.

Correct Answer. c

(125). Treatment options for stage I nonseminomatous germ cell tumor (NSGCT) include all of the following, EXCEPT:

a. Surveillance or "watchful waiting"

b. Retroperitoneal lymph node dissection (RPLND)

c. Adjuvant chemotherapy

d. High-dose chemotherapy

Solution. (d) High-dose chemotherapy


Ref: Read the text below.
Sol:
- Surveillance is preferred by many doctors and patients because the vast majority of patients can be cured when they relapse.
- However, successful surveillance requires strict compliance on behalf of both doctor and patient. RPLND is the most accurate way to
stage patients, and cure rates in expert hands approach 100%, but it is associated with potentially significant morbidity.
- Chemotherapy, usually with 2 cycles of bleomycin, etoposide, and cisplatin, is also associated with an excellent outcome, but there are
concerns regarding longer-term toxicities, such as infertility and second cancers.
- High-dose chemotherapy with autologous stem cell support is feasible and potentially cures a proportion of patients with resistant
disease.
- However, data on this treatment as initial therapy for higher-risk patients are scarce.

Correct Answer. d

Copyright © 2014 Delhi Academy of Medical Sciences, All Rights Reserved. 63/94
(126). Primary effusion lymphoma is characterized by:

a. Usually presenting as serous effusions without detectable tumor masses

b. Association with HHV-8/ Kaposi sarcoma virus

c. Majority occur in setting of HIV infection

d. All of above

Solution. (d) All of above


Ref:Read the text below
Sol :
Primary effusion lymphoma
- PEL is a neoplasm of large B-cells usually presenting as serous effusions without detectable tumor masses
- It is universally associated with HHV-8/ KSHV, most often occurring in the setting of Immunodeficiency
- Majority of cases occur in setting of HIV infection
- Most patients are young to middle aged homosexual males
- Sites of involvement- most common sites are pleural, pericardial & peritoneal cavities
- Typically only one body cavity is involved
- Other sites of involvement include GIT, soft tissue & other extranodal sites
- Patients typically present with effusions in absence of lymphadenopathy or organomegaly
- Most patients are co-infected with EBV
- Immunophenotype: lymphoma cells usually express leukocyte common antigen (CD45) but are usually negative for pan-B cell markers
such as CD19, CD20
- Activation and plasma cell-related markers such as CD30, CD38, & CD138 are usually demonstrable
- Despite the usual presence of EBV, staining for LMP-1 is negative
- Postulated cell of origin- post- germinal center B-cell

Correct Answer. d

(127). The preferred modality of treatment for Stage II colon cancer is?

a. Surgery

b. Surgery + chemotherapy

c. Chemotherapy + radiation

d. Radiation

Solution. (b) Surgery + chemotherapy


Ref:Read the text below
Sol :

Correct Answer. b

Copyright © 2014 Delhi Academy of Medical Sciences, All Rights Reserved. 64/94
(128). Which of the following is not a cyclin dependent kinase (CDK) inhibitor

a. p21

b. p27

c. p53

d. p57

Solution. (c) p53


Ref.:Robbin’s - 285
Sol :
- P53 is a tumor suppressor gene, that control some CDK inhibitors like p21, but p53 itself is not a CDK inhibitor.
Cell cycle inhibitors
- The activity of cyclin – CDK complexes is tightly regulated by inhibitors, called CDK inhibitors.
- By inhibiting CDK they inhibit cell cycle and replication – act as tumor suppressors.
There are two main classes of CDK inhibitors –
- The Cip/kip family
- The INK4/ARF family
The Clip/kip family
- It has 3 components, P21, p27 and p57.
- This binds and inactivate cyclin – CDK complexes.
- Transcriptional activation of p21 is under the control of p53, a tumor suppressor gene that is mutated in a large proportion of human
cancers.

Correct Answer. c

(129). In the mitogen activated protein kinase pathway, the activation of RAS is counteracted by –

a. Protein kinase C

b. GTPase activating protein

c. Phosphatidyl inositol

d. Inositol triphosphate

Solution. (b) GTPase activating protein


Ref.:Robbin’s - 282
Sol :
- RAS protein is a signal transducing protein.
- Signal transducing proteins receive growth signals from outside the cell and transmit it to the nucleus.
- Normally RAS proteins are attached to the cytoplasmic aspect of the plasma membrane where they keep on converting between an
active signal transmitting state and an inactive quiescent state.
- In the inactive state the RAS proteins bind Guanosine diphosphate (GDP).
- When the cells are stimulated by growth factors or other receptor – ligand interactions, RAS becomes activated by exchanging GDP for
GTP.
- This activates RAS then recruits raf – 1 and activates MAP – kinase pathway.
- The MAP kinases so activated target nuclear transcription factors and then they promote mitogenesis.

Correct Answer. b

Copyright © 2014 Delhi Academy of Medical Sciences, All Rights Reserved. 65/94
(130). Hairy cell leukemia commonly presents as

a. Infections

b. Bleeding

c. Splenomegaly

d. Paraproteinemia

Solution. (c) Splenomegaly


Ref.:Read the text below
Sol :
- In hairy cell leukemia, the "hairy cells" (malignant B lymphocytes) accumulate in the bone marrow, interfering with the production of
normal white blood cells, red blood cells, and platelets. Consequently, patients may develop infections related to low white blood cell
count, anemia and fatigue due to a lack of red blood cells, or easy bleeding due to a low platelet count.
-Leukemic cells may gather in the spleen and cause it to swell; this can have the side effect of making the person feel full even when he
or she has not eaten much.
- Hairy cell leukemia is commonly diagnosed after a routine blood count shows unexpectedly low numbers of one or more kinds of normal
blood cells
- Platelet function may be somewhat impaired in HCL patients, although this does not appear to have any significant practical effect.
- Patients with a high tumor burden may also have somewhat reduced levels of cholesterol,especially in patients with an enlarged spleen.
- Cholesterol levels return to more normal values with successful treatment of HCL

Correct Answer. c

(131). Translocation in Philadelphia chromosome

a. t(8,14)

b. t(9,22)

c. t(8,22)

d. t(9,11)

Solution. (b) t(9,22)


Ref.:Read the text below
Sol :

Correct Answer. b

Copyright © 2014 Delhi Academy of Medical Sciences, All Rights Reserved. 66/94
(132). Most common leukemia in children

a. ALL

b. AML

c. CLL

d. CML

Solution. (a) ALL


Ref.:Read the text below
Sol :
- Neoplasms of immature B (pre-B) and T (pre-T) cells which are referred to as lymphoblasts
- 85% are pre-B cell tumors and present in childhood as leukemias
- Less common pre-T cell tumors present in adolescent males as lymphomas with thymomas
- Most common cancer in childhood (2nd most common – brain tumors)

Correct Answer. a

(133). Tumorogenesis in aging is due to

a. Telomerase reactivation

b. Telomerase inactivation

c. Increased apoptosis

d. Suppression of proto-oncogenes

Solution. (a) Telomerase reactivation


Ref.:Robbin’s - 296
Sol :
- Telomerase is a reverse transcriptase and is responsible for telomer synthesis and maintaining the length of telomer.
- It is RNA dependent DNA polymerase. It has one segment that is complimentary to TTAGGG repeat and is used as template for the
replication of telomeric sequence.
- This enzyme is absent from most somatic cells and hence they suffer progressive loss of telomers and they exit the cell cycle.
Telomerase is present in Germ cells thus explaining the ability of these cells to self replicate extensively.
- If loss of telomere is the basis of finite life span then how do cancer cells continue to divide actively ???
- In cancer cells telomerase activity is reactivated. Telomerase activity has been detectd in more than 90% of human tumors.
- Thus telomerase activity and maintenance of telomere length are essential for the replicative potential in cancer cells.

Correct Answer. a

(134). All of the following constitute familial cancer syndrome except –

a. Xeroderma-pigmentosum

b. Retinoblastoma

c. Neurofibromatosis

d. MEN-1

Solution. (a) Xeroderma-pigmentosum


Ref.:Robbin’s 8th/e p. 275
Sol :
- All are familial cancer syndromes.
- Xeroderma-pigmentosa is autosomal recessive, defective DNA repair syndrome.
- Other three are autosomal dominant inherited cancer syndrome.
- Amongst the given options best answer is a i.e. xerodermapigmentosa because autosomal dominant inherited cancer syndromes are the
most common type of genetically predisposed cancer.

Correct Answer. a

Copyright © 2014 Delhi Academy of Medical Sciences, All Rights Reserved. 67/94
(135). The following parasitic infections predispose to malignancies –

a. Pargibunuasus

b. Clonorchiasis

c. Schistosomiasis

d. B and C

Solution. (d) B and C


Ref.: Robbin’s - 239
Sol : Parasites associated with malignancy
- Clonorchissinensis → Liver, bile duct (cholangiocarcinoma) and Pancreas (adenocarcinoma)
- Opisthorchis viverrini → Bile duct (chalangio carcinoma)
- Schistosoma haematobium → Urinary bladder (Squamous cell ca)
- Fasciola hepatica → Cholangiocarcinoma

Correct Answer. d

(136). The most important prognostic indicator of mesenchymal tumour is

a. Type of primary

b. Grade

c. Size

d. Site

Solution. (b) Grade


Ref.:Robbin’s - 1219
Sol :
- Whatever the type, the grade of a soft tissue sarcoma is most important for predicting its behavior.
- Grading is base on the degree of differentiation of tumor cells and the number of mitosis within the tumor.

Correct Answer. b

(137). Multiple myeloma is diagnosed by

a. Bence Jones proteinuria

b. Bone marrow plasma cells > 30%

c. Lytic bone lesions

d. All the above.

Solution. (d) All the above.


Ref.:Read the text below
Sol :

Correct Answer. d

Copyright © 2014 Delhi Academy of Medical Sciences, All Rights Reserved. 68/94
(138). Pathognomic feature of Burkitt’s lymphoma

a. Russel bodies

b. Reed Stenberg cells

c. Starry sky pattern

d. Howell-Jolly bodies

Solution. (c) Starry sky pattern


Ref.:Read the text below
Sol :
- The Burkitt’s lymphoma consists of sheets of a monotonous (i.e. similar in size and morphology) population of medium size lymphoid
cells with high proliferative activity and apoptotic activity.
- The "starry sky" appearance seenunder low power is due to scattered tingible body-laden macrophages (macrophages containing dead
body of apoptotic tumor cells).
-The old descriptive term of "small non-cleaved cell" is misleading. The tumor cells are mostly medium in size (i.e. tumor nuclei size
similar to that ofhistiocytes or endothelial cells).
- "Small non-cleaved cells" are compared to "large non-cleaved cells" of normal germinal center lymphocytes.
- Tumor cells possess small amount of basophilic cytoplasm. The cellular outline usually appears squared off.

Correct Answer. c

(139). Most common tumor associated with pure red cell aplasia

a. Hepatoma

b. Hodgkins lymphoma

c. Thymoma

d. Bronchogenic carcinoma

Solution. (c) Thymoma


Ref.:Read the text below
Sol :
- Marked hypoplasia of marrow erythroid precursors in the setting of normal granulopoiesis and thrombopoiesis
- 1° or 2° neoplasms (thymomas, large granulocytic leukemia), autoimmune or parvovirus infections
- Drugs : phenytoin, azathioprine, chloramphenicol, procainamide, isoniazid, erythropoietin
- In most of the patients resection fothymic tumor is followed by hematologic improvement
- Plasmapheresis used in refractory cases.

Correct Answer. c

(140). The incidence of which of the following cancers has increased in the past 4 decades

a. Carcinoma of pancreas

b. Carcinoma of stomach

c. Carcinoma of lung

d. Carcinoma of colon

Solution. (c) Carcinoma of lung


Ref.:Robbin’s - 283
Sol :
- Incidence of lung carcinoma has increased in both male & female during last 4-5 decades.
- Incidence of stomach carcinoma has decreased.

Correct Answer. c

Copyright © 2014 Delhi Academy of Medical Sciences, All Rights Reserved. 69/94
(141). Choristoma is

a. Normal tissue in abnormal site

b. Normal tissue in excess at normal site

c. Abnormal tissue at any site

d. None of the above

Solution. (a) Normal tissue in abnormal site


Ref.:Robbin’s - 262
Sol :
- An ectopic mass of normal tissue is sometimes called choriostoma e.g. Rest of adrenal cells under the kidney capsule.

Correct Answer. a

(142). Which of the following functions is associated with the oncogenic L ‘MYC’ in lung cancer –

a. Proteine kinase

b. GTP binding protein

c. Nuclear binding protein

d. Growth factor

Solution. (c) Nuclear binding protein


Ref.:Robbin’s 8th/e p. 281
Sol :
- MYC is a protein, activities attributed to MYC are – Histone acetylation, Reduced cell adhesion, increased cell motility, increased
protein synthesis and decreased proteinase activity.

Correct Answer. c

(143). A child is born with a single functional copy of a tumor suppressor gene. At the age of 5 years, the remaining normal allele is lost through
mutation. As a result, the ability to control the transition from G1 to the S phase of the cell cycle is lost. Which of the following neoplasms
is most likely to arise by means of this mechanism

a. Retinoblastoma

b. Breast carcinoma

c. Adenocarcinoma of colon

d. Cerebral astroctoma

Solution. (a) Retinoblastoma


Ref.:Robbin’s -289
Sol :
Heriditory retinoblastoma
- One genetic change (first hit) in RB gene is inherited from an affected parent, where as second mutation (second hit) occurs in
postnatal life – Both allel of RB are lost.
Sporadic retinoblastoma :
- Both multatons (first & second hits) occur postnataly.
- Loss of both the normal allel of RB gene will result in loss of regulatory mechanism at G1/S transition, as RB, in its hypophosphorylated
form, blocks cell cycle at G1/S transition

Correct Answer. a

Copyright © 2014 Delhi Academy of Medical Sciences, All Rights Reserved. 70/94
(144). Keratin is a tumour marker for

a. Carcinoma cervix

b. Neurofibroma

c. Rhabdoyosarcoma

d. Chorio carcinoma

Solution. (a) Carcinoma cervix


Ref.:Robbin’s - 324
Sol :
- Presence of cytokerotinsdelected by immunohistochemistry points to an epithelial origin (carcinoma).

Correct Answer. a

(145). Hodgkins lymphoma with best prognosis

a. Mixed cellularity

b. Lymphocyte depletion

c. Lymphocyte rich

d. Nodular sclerosis

Solution. (d) Nodular sclerosis


Ref.:Read the text below
Sol :

Correct Answer. d

Copyright © 2014 Delhi Academy of Medical Sciences, All Rights Reserved. 71/94
(146). The single most important predictor of survival in multiple myeloma

a. IL-6

b. Bence jones proteinuria

c. CD 138 positivity

d. Serum β2-microglobulin

Solution. (d) Serum β2-microglobulin


Ref.:Read the text below
Sol :

Correct Answer. d

(147). In exocrine pancreatic cancer, the commonest tumour marker to be elevated as –

a. CA-125

b. CEA

c. CA-15-3

d. CA-19-9

Solution. (d) CA-19-9


Ref.:Robbin’s - 587
Sol :
- No tumor-specific markers exist for pancreatic cancer; markers such as serum cancer antigen (CA) 19-9 have low specificity.
- Most patients with pancreatic cancer will have an elevated CA 19-9 at diagnosis.
-Following or during definitive therapy, the increase of CA 19-9 levels may identify patients with progressive tumor growth.

Correct Answer. d

Copyright © 2014 Delhi Academy of Medical Sciences, All Rights Reserved. 72/94
(148). Major contribution to cachexia with advanced cancer

a. Clathrin

b. Histamine

c. Interferon

d. Turnor – necrosis-factor (TNT)

Solution. (d) Turnor – necrosis-factor (TNT)


Ref.:Robbin’s 8th/e p. 320
Sol :
- Patients with cancer commonly suffer progressive loss of body fat which is accompanied by profound weakness anorexia, and anemia.
This wasting syndrome is referred to as cachexia.
- TNF produced by macrophages or some tumor cells is the most important mediator of cachexia.

Correct Answer. d

(149). Carcinogenesis attributed to the formation of pyramidine dimmers in DNA commonly occurs following exposure to –

a. UV radiation

b. Infra-red radiation

c. Viral oncogenesis

d. Chemical carcinogens

Solution. (a) UV radiation


Ref.:Robbin’s - 312
Sol :
- UV rays damage DNA, resulting in the formation of pyrimidine dimmers between adjacent pyrimidines on the same DNA.
- Other DNA damages include, formation of pyrimidine- pyrimidine photoproducts, single strand break, and DNA protein cross links.

Correct Answer. a

(150). Following are spontaneous resolution tumours except

a. Malignant melanoma

b. Osteogenic sarcoma

c. Cholangio carcinoma

d. Retinoblastoma

Solution. (b) Osteogenic sarcoma


Ref.:Robbin’s - 214
Sol :
Due to immune surveillance some tumours may undergo spontaneous regression Example are –
- Neuroblastoma
- Malignant melanoma
- Retinoblastoma

Correct Answer. b

Copyright © 2014 Delhi Academy of Medical Sciences, All Rights Reserved. 73/94
(151). Characteristic translocation in follicular lymphoma

a. t(14;18)

b. t(15;17)

c. t(9;22)

d. t(14;22)

Solution. (a) t(14;18)


Ref.:Read the text below
Sol :

Correct Answer. a

Copyright © 2014 Delhi Academy of Medical Sciences, All Rights Reserved. 74/94
(152). CD5 is expressed in

a. Mantle cell lymphoma

b. Chronic myeloid lymphoma

c. Follicular lymphoma

d. Burkitt lymphoma

Solution. (a) Mantle cell lymphoma


Ref.:Read the text below
Sol :

Correct Answer. a

(153). Small cell lung carcinoma can cause all of the following except:

a. Hypercalcaemia

b. Cushing’s syndrome

c. Clubbing

d. Lambert Eaton Myasthenic syndrome

Solution. (a) Hypercalcaemia


Reference – Read the text below
Sol
- Hypercalcaemia of malignancy can be a paraneoplastic syndrome resulting from PTH related peptide or PTH (humoral hypercalcaemia
of malignancy HHM) or can result from local paracrine factors from tumours within bone (local osteolytic hypercalcaemia LOH).
- It is commonly associated with NSCLC, breast cancer, renal cell carcinoma, head & neck cancers, bladder cancer & myeloma, but not
with small cell lung cancers.
- The other conditions are common paraneoplastic syndromes associated with small cell lung cancer.

Correct Answer. a

Copyright © 2014 Delhi Academy of Medical Sciences, All Rights Reserved. 75/94
(154). A 45-year-old man is seen with 4 weeks of progressive dysphagia that now includes semisolids. He is having constant chest discomfort
and cough and has noted a 10 kgs weight loss in the last 6 weeks. A chronic smoker and a 6-pack-a-week drinker, he has no other
significant health problems. Which of the following is the best initial diagnostic study ?

a. MRI

b. Barium swallow

c. Esophagogastroduodenoscopy (EGD)

d. Endoscopic retrograde cholangiopancreatography (ERCP)

Solution. (d) Endoscopic retrograde cholangiopancreatography (ERCP)


Ref: Read the text below.
Sol:
- All the choices, with the exception of the ERCP, are good ones to evaluate this patient for esophageal carcinoma or to determine its
spread, but the best initial test is the EGD.
- The symptoms of progressive dysphagia, chest pain, cough and weight loss should cause alarm bells to ring.
- The fastest and most definitive answers can be obtained with an EGD.

Correct Answer. d

(155). A 67-year-old man complains of shortness of breath on exertion and right-sided chest pain.The pain is constant in nature; he has no fever
or chills and no sputum production. On examination,air entry to the left lower lobe is reduced and the area is dull on percussion. The
CXR reveals left lower lobe pleural effusion,pleural thickening with calcification (plaques), and lower lobe fibrosis. Further history from
the patient is significant in that he is a lifetime nonsmoker, and that he worked as a pipe fitter until retiring 2 years ago. Which of the
following is the most likely diagnosis?

a. Adenocarcinoma

b. Squamous cell carcinoma

c. Lymphoma

d. Mesothelioma

Solution. (d) Mesothelioma


Ref: Read the text below
Sol :
- The history of being a pipe fitter suggests asbestos exposure, and the CXR finding of pleural plaques and lower lobe fibrosis confirms
prior asbestos exposure.
- The classic associated cancer is mesothelioma.
- However, in 30–50% of cases, no history of asbestos exposure is apparent. The average age on presentation is 60, and this is typically
many years after the exposure.
- The most common presenting symptoms are dyspnea and nonpleuritic chest pain.
- Smoking injury in patients with asbestos exposure increases the risk of other lung cancers such as adenocarcinoma or squamous cell
carcinoma.

Correct Answer. d

(156). Which of the following statements about bone sarcoma is correct?

a. Distal bone sarcomas have a better prognosis

b. Lung metastases are a late sign

c. Local lymph node involvement is very common

d. Articular cartilage is a common plane for tumor spread

Solution. (a) Distal bone sarcomas have a better prognosis


Ref: Read the text below
Sol :
- Overwhelmingly, the major prognostic factor in osteosarcoma is location of the tumor.
- Pelvic and axial lesions do worse than those in the extremities, and survival is better in tibial tumors than femoral tumors.
- Lung metastases are very common.

Correct Answer. a

Copyright © 2014 Delhi Academy of Medical Sciences, All Rights Reserved. 76/94
(157). A 4-yr-old boy has a bone marrow biopsy that shows 4% blasts. No blasts are seen on peripheral blood smear. Findings on physical examination and
other laboratory studies are normal. The most likely diagnosis is:

a. Acute lymphoblastic leukemia

b. Acute myelogenous leukemia

c. Chronic myelogenous leukemia

d. Preleukemia syndrome

Solution. (d) Preleukemia syndrome


Ref: Read the text below
Sol:
- ALL is diagnosed by a bone marrow evaluation that demonstrates more than 25% of the bone marrow cells as a homogeneous population of
lymphoblasts.

Correct Answer. d

Copyright © 2014 Delhi Academy of Medical Sciences, All Rights Reserved. 77/94
(158). A breakthrough treatment for chronic myelogenous leukemia in patients with the BCR-ABL chromosomal translocation is:

a. Imatinib

b. Umbilical cord blood transplantation

c. Cis-retinoic acid

d. Timed intensive induction chemotherapy

Solution. (a) Imatinib


Ref: Read the text below
Sol:
- Imatinib was designed specifically to inhibit BCR-ABL tyrosine kinase.
Uses Clinical Chronic myelogenous leukemia
- Imatinib is used in chronic myelogenous leukemia (CML), gastrointestinal stromal tumors (GISTs) and a number of other malignancies.
One study demonstrated that imatinibmesylate was effective in patients with systemic mastocytosis, including those who had the D816V
mutation in c-Kit.
- Experience has shown, however, that imatinib is much less effective in patients with this mutation, and patients with the mutation
comprise nearly 90% of cases of mastocytosis. Early clinical trials also show its potential for treatment of hypereosinophilic syndrome
and dermatofibrosarcoma protuberans
- Imatinib has been shown to be more effective than the previous standard treatment of α-interferon and cytarabine.
Plexiform neurofibromas
- For treatment of progressive plexiform neurofibromas associated with neurofibromatosis type I, early research has shown potential for
using the c-kit tyrosine kinase blocking properties of imatinib.
- The most common side effects include weight gain, reduced number of blood cells (neutropenia, thrombocytopenia, anemia), headache,
edema, nausea, rash, and musculoskeletal pain.
- Severe congestive cardiac failureis an uncommon but recognized side effect of imatinib and mice treated with large doses of imatinib
show toxic damage to their myocardium

Correct Answer. a

(159). Which of the following laboratory findings is common in a patient with an esophageal carcinoma ?

a. Acidosis

b. Hypoglycemia

c. Hypercalcemia

d. Macrocytic, hyperchromic anemia

Solution. (c) Hypercalcemia


Ref: Read the text below.
Sol:
- Hypercalcemia is a common initial presentation, maybe even before dysphagia.
- Patients will complain of rapid onset of spreading weakness, nausea, sedation, and eventually stupor.
- The anemia is uauslly one associated with blood loss, a microcytic-hypochromic anemia.
- With metastasis, liver enzymes will rise, but usually not markedly.

Correct Answer. c

(160). Which of the following is not a source of structural dysphagia?

a. Scleroderma

b. Chagas disease

c. Zenker’s diverticulum

d. Mediastinal lymphadenopathy

Solution. (a) Scleroderma


Ref: Read the text below.
Sol:
- Scleroderma is a motor disorder that will cause some esophageal dysphagia or changes in up to 80 percent of scleroderma patients.
- The dysphagia is due to decreased or absent peristalsis.
- Chagas disease can cause a megaesophagus secondary to esophageal infestation by trypanosomiasis.

Correct Answer. a

Copyright © 2014 Delhi Academy of Medical Sciences, All Rights Reserved. 78/94
(161). A 14-yr-old boy presents with a bony, nonpainful mass in the distal femur that has been slowly enlarging. There are no associated
symptoms. Study the microscopic pathology in Fig The mass is most likely to represent which of the following tumors?

a. Ewing sarcoma

b. Osteosarcoma

c. Langerhans cell histiocytosis

d. Osteochondroma

Solution. (d) Osteochondroma


Ref: Read the text below
Sol:
- Osteochondroma (exostosis) is one of the most common benign bone tumors in children.
- Many of these tumors are asymptomatic or are detected when the child or the parent notes a bony, nonpainful mass.
- Most arise in the metaphyses of long bones.

Correct Answer. d

(162). Which of the following is the most common neoplasm involving the liver in children?

a. Hepatocellular carcinoma

b. Hepatoblastoma

c. Wilms tumor

d. Rhabdomyosarcoma

Solution. (b) Hepatoblastoma


Ref: Read the text below
Sol:
- Primary hepatic tumors are rare in children, accounting for 1% of malignancies in childhood.
- Approximately two thirds of hepatic tumors in children are hepatoblastomas.

Correct Answer. b

Copyright © 2014 Delhi Academy of Medical Sciences, All Rights Reserved. 79/94
(163). An infant has a hemangioma that grows rapidly in size, leading to thrombocytopenia and microangiopathic hemolytic anemia. Which of the
following terms describes this condition?

a. Kaposi-like form of infantile hemangioma

b. Albright syndrome

c. Kasabach-Merritt syndrome

d. Maffucci syndrome

Solution. (c) Kasabach-Merritt syndrome


Ref: Read the text below
Sol:

Correct Answer. c

(164). Study the Fig The most common presenting sign of given tumor is:

a. Heterophoria

b. Hypopyon

c. Leukocoria

d. Coloboma

Solution. (c) Leukocoria


Ref: Read the text below
Sol:
- The clinical manifestations of retinoblastoma vary, but the initial sign in the majority of cases is leukocoria (white pupillary reflex, also
known as "cat eye") instead of the normal red pupillary reflex.

Correct Answer. c

Copyright © 2014 Delhi Academy of Medical Sciences, All Rights Reserved. 80/94
(165). Which of the following is not considered a risk factor for the development of esophageal carcinoma?

a. Alcohol abuse

b. Tobacco abuse

c. Zenker’s diverticulum

d. Plummer-Vinson syndrome

Solution. (c) Zenker’s diverticulum


Ref: Read the text below.
Sol:
- Zenker’s diverticulum is a mechanical problem usually associated with the upper esophagus.
- A diverticulum develops and can enlarge with ingested foods.
- It causes regurgitation and cough but it does not predispose the esophagus to cancer.
- Smoking, alcohol, and chemical ingestion can cause tissue disruption or decreased blood supply.

Correct Answer. c

(166). First-line chemotherapy of choice for advanced epithelial ovarian cancer is :

a. Cisplatin, paclitaxel

b. Cyclophosphamide, hexamethylmine, doxorubicin, cisplatin (CHAP)

c. Melphalan

d. Cisplatin, cyclophosphamide

Solution. (a) Cisplatin, paclitaxel


Ref: Read the text below.
Sol:
- Based on two large prospective trials, the combination of cisplatin plus paclitaxel (Taxol) is considered the treatment of choice for
stages III and IV epithelial ovarian cancer.
- These trials compared this treatment to the former standard regimen of cisplatin and cyclophosphamide, and the survival of the
patients who received the paclitaxelcontaining treatment was longer. Current studies are comparing carboplatin and paclitaxel versus
cisplatin and paclitaxel. The use of these agents in low-stage disease is also being tested.

Correct Answer. a

(167). The recommended screening test(s) in the general population for colorectal cancer is/are

a. Carcinoembryonic antigen (ECA) annually over age 50

b. Annual stool tests for occult blood in persons over 50

c. Flexible sigmoidoscopy every 3 to 5 years for individuals over 50

d. B and C

Solution. (d) B and C


Ref: Read the text below.
Sol:
- Currently, the CEA is not recommended as a screening test.
- Annual test for occult blood and interval flexible sigmoidoscopy are recommended by the American Cancer Society for individuals over
50 without other known risk factors.

Correct Answer. d

Copyright © 2014 Delhi Academy of Medical Sciences, All Rights Reserved. 81/94
(168). The average survival of a patient who has non-resectable pancreatic cancer is

a. 6 months

b. 18 months

c. 2 to 3 years

d. 3 to 5 years

Solution. (a) 6 months


Ref: Read the text below.
Sol:
- Pancreatic cancer is an extremely aggressive and deadly disease.
- Even patients with resectable disease who undergo surgery have a 5-year survival rate of less than 25 percent.

Correct Answer. a

(169). Most breast cancers are

a. Lobular

b. Ductal

c. Inflammatory

d. Other special histologies

Solution. (b) Ductal


Ref: Read the text below.
Sol:
- Mammary ductal carcinoma is the most common type of breast cancer in women. It comes in two forms: invasive ductal carcinoma
(IDC), an infiltrating, malignant and abnormal proliferation of neoplastic cells in the breast tissue, or ductal carcinoma in situ (DCIS), a
noninvasive, possibly malignant, neoplasm that is still confined to the milk ducts (lactiferous ducts), where breast cancer most often
originates.
- Inflammatory lesions account for 1 percent of breast cancers, and lobular lesions are found in 9 percent of breast cancers.
- Special histologies are found approximately 12 percent of the time.

Correct Answer. b

(170). The Growth Fraction is defined as :

a. The fraction of growth over a specific interval

b. The number of tumors that actively grow within a month

c. The number of cells in the tumor mass that are actively undergoing cell division

d. The number of tumors that are actively affected by an increase in caloric intake

Solution. (c) The number of cells in the tumor mass that are actively undergoing cell division
Ref: Read the text below.
Sol:
- The two major factors that affect the rate at which tumors grow are the growth fraction and cell death.
- The growth fraction represents the actual number of cells that are actively dividing, and hence, are susceptible to the effects of those
chemotherapeutic agents that interfere with cell division.
- The concept is important because some tumors have a low growth fraction, for example, and are therefore less likely to be affected by
the use of some chemotherapeutic drugs.

Correct Answer. c

Copyright © 2014 Delhi Academy of Medical Sciences, All Rights Reserved. 82/94
(171). Methotrexate is an drug which is :

a. Cell cycle specific, proliferation dependent

b. Cell cycle specific, less proliferation dependent

c. Cell cycle nonspecific, proliferation dependent

d. Cell cycle nonspecific, less proliferation dependent.

Solution. (b) Cell cycle specific, less proliferation dependent


Ref: Read the text below.
Sol:
- The antimetabolite, methotrexate, is a cell cycle specific drug because its primary mode of action requires cell division.
- It is not dependent on the proliferative capacity of the tissue involved.

Correct Answer. b

(172). Conventional external beam radiation therapy is usually given in multiple daily fractions of 180 to 200 cGy per fraction. Why is radiation
fractionated over many treatments as opposed to being given in one or two much larger treatments ?

a. A larger number of cancer cells are eradicated with fractionated treatment.

b. Fractionation allows repair of normal tissues and decreases complication.

c. Fractionation allows reoxygenation which improves radiation sensitivity.

d. All of above.

Solution. (b) Fractionation allows repair of normal tissues and decreases complication.
Ref: Read the text below.
Sol:
- Fractionated radiation therapy eradicates less cells than the same total dose when given in a single treatment.
- Fractionation of radiation allows the repair of normal tissues between treatments and thus decreases complications.
- It additionally allows for hypoxic tumors to reoxygenate and become more sensitive to radiation.

Correct Answer. b

Copyright © 2014 Delhi Academy of Medical Sciences, All Rights Reserved. 83/94
(173). A 58-year-old, obese nulliparous woman with diabetes mellitus and a history of irregular menses most of her life until menopause at age 55 is at
increased risk for the development of endometrial cancer. The factor which places her at greatest risk is :

a. Obesity

b. Nulliparity

c. Diabetes mellitus

d. History of irregular menses

Solution. (a) Obesity


Ref: Read the text below.
Sol:

Correct Answer. a

(174). Abdominal perineal resection is done in colorectal Ca on the basis of :

a. Age of patient

b. Distance from anal verge

c. Fixity of tumor

d. Hepatic metastasis

Solution. (b) Distance from anal verge


Ref.:Devita - 1113
Sol :
The principal indication for AP resection is a rectal carcinoma situated in the distal (lower) one-third of the rectum.[1] Other indications
include recurrent or residual anal carcinoma (squamous cell carcinoma) following initial, usually definitive combination
chemoradiotherapy.
APRs involves removal of the anus, the rectum and part of the sigmoid colon along with the associated (regional) lymph nodes, through
incisions made in the abdomen and perineum. The end of the remaining sigmoid colon is brought out permanently as an opening, called a
colostomy, on the surface of the abdomen.
Abdominal perineal resection is done in colorectal Ca on the basis of distance from anal verge

Correct Answer. b

Copyright © 2014 Delhi Academy of Medical Sciences, All Rights Reserved. 84/94
(175). When carcinoma of stomach develops secondarily to pernicious anemia, it is usually situated in the :

a. Prepyloric region

b. Pylorus

c. Body

d. Fundus

Solution. (d) Fundus


Ref.:Sleisenger and Fordrans text book of Gastrointestinal disease - 813
Sol :
- “Diffuse corporal atrophic gastritis (DCAG) is an autoimmune destruction of funduc glands. It is the pathologic process seen in patients
with pernicious anemia.
- It exhibits achlorhydria or hypochlorhydria, hypergastrenemia, antral G cell hyperplasia, circulating parietal cell and intrinsic factor
antibodies.”
- The incomplete (colonic) type of intestinal metaplasia also occur in DCAG. Atrophic glands with extensive intestinal metaplasia are
confined to fundus and are risk factor for gastric carcinoma.
- Chronic atrophic gastritis can be classified as autoimmune (involves proximal stomach), Hypersecretory (distal stomach) and
environmental (at multiple random areas at junction of oxynticantral mucosal)

Correct Answer. d

(176). Commonest malignancy of the small intestine is :

a. Leiomyoma

b. Lymphoma

c. Adenocarcinoma

d. Hemangioma

Solution. (c) Adenocarcinoma


Ref.:Sabiston- 1100
Sol :

Correct Answer. c

(177). After undergoing surgery, for carcinoma of colon a patient developed single liver metastasis of 2 cm, what would be the next line of
management :

a. Resection

b. Chemoradiation

c. Acetic acid injection

d. Radiofrequency ablation

Solution. (a) Resection


Ref.:Bailey & Love 25/e 1184
Sol :
Treatment of hepatic metastasis (< 3 in one lobe of liver)
- It is not a contraindication for resection of tumor.
- After recovery from colonic resection, hepatic resection is done (resection can also be done in lung metastasis). Most wait 12 weeks
before restaging.
- In case of irresectable symptomatic hepatic metastasis (jaundice, ascites, hepatomegaly); local ablative treatment (cryosurgery, radio-
frequency or microwave coagulation, embolisation, hepatic intra-arterial chemotherapy) is indicated.

Correct Answer. a

Copyright © 2014 Delhi Academy of Medical Sciences, All Rights Reserved. 85/94
(178). The tendency of colonic carcinoma to metastasize is best assessed by :

a. Size of tumor

b. Carcinoembryogenic antigen (CEA) levels

c. Depth of penetration of bowel wall

d. Proportion of bowel circumference involved.

Solution. (b) Carcinoembryogenic antigen


Ref.:Sabiston- 1100
Sol :
- “When used alone CEA assay is 70% accurate in predicting the development of liver metastases within 1 year; in combination with CT
scans, accuracy increases upto 90%”
CEA level is increased in : —
- Tumor recurrence
- Metastases from colorectal cancer

Correct Answer. b

(179). Prolonged exposure to para-aminobiphenyl is associated with the development of

a. Urinary bladder carcinoma

b. Lung carcinoma

c. Large-bowel carcinoma

d. Pancreatic carcinoma

Solution. (a) Urinary bladder carcinoma


Ref: Read the text below
Sol :
- Carcinoma of the urinary bladder has been linked to several predisposing factors.
- The development of bladder carcinoma is associated with the ingestion of chemicals that are excreted in urine as conjugated
orthoaminophenols; among these chemicals are beta-naphthylamine and para-aminobiphenyl.
- Chronic schistosome infection and vesicle calculus also have been identified as predisposing factors.

Correct Answer. a

(180). All the following statements concerning adenocarcinoma of the small intestine are true except

a. It is the most frequent primary malignant small-bowel tumor

b. Presenting features are those of intestinal obstruction

c. Its incidence is higher in persons with crohn’s disease

d. The prognosis is worse, stage for stage, than with colonic carcinoma

Solution. (d) The prognosis is worse, stage for stage, than with colonic carcinoma
Ref: Read the text below
Sol :
- Despite the large surface area of the small-bowel mucosa, the incidence of adenocarcinoma of the small bowel, the most frequent
neoplasm of the small bowel, is rare.
- However, the incidence is higher in persons with immunoglobulin A deficiency or Crohn’s disease.
- Presenting features are signs and symptoms of small-bowel obstruction.
- Prognosis is poor, because the disease is usually difficult to identify. Stage for stage, the survival rates of colonic carcinoma and small-
bowel carcinoma are no different.

Correct Answer. d

Copyright © 2014 Delhi Academy of Medical Sciences, All Rights Reserved. 86/94
(181). The most common site of gastrinoma is the

a. Gastric antrum

b. Duodenum

c. Pancreas

d. Spleen

Solution. (c) Pancreas


Ref: Read the text below
Sol :
- Gastrinoma produces Zollinger-Ellison syndrome, which is associated with markedly elevated gastric acid secretion and ulcer disease of
the upper gastrointestinal tract. The most common site of occurrence is the pancreas.
- However, gastrinoma has been known to occur in the gastric antrum, duodenum, spleen, and ovary. Removal of the gastrinoma can
result in a cure.
- A thorough search must be made at surgical exploration to locate the tumor, which in early stages will be small.
- The gastrinoma triangle is defined as the junction of the cystic and common bile ducts, second and third portion of the duodenum, and
the division of the pancreatic neck and body. 90% of gastrinomas are located here.

Correct Answer. c

(182). Which one of the following is an apoptosis inhibitor

a. P53

b. BCL2

c. Rb

d. C-myc

Solution. (b) BCL2


Ref: Read the text below
Sol :
- Growth factors and other survival signals stimulate production of anti-apoptotic proteins the main ones being Bcl-2, Bcl-x , and Mcl-1.

Correct Answer. b

(183). All are anit-apoptotic members of the bcl-2 family except

a. Bad

b. Bcl xL

c. Bcl 2

d. Mcl

Solution. (a) Bad


Ref: Read the text below
Sol :
- Bad and bid are pro-aproptotic members When cells are deprived of survival signals or their DNA is damaged, or misfolded proteins
induce ER stress, sensors of damage or stress are activated.
- These sensors are also members of the Bcl family, and they include proteins called Bim, Bid, and Bad that contain a single “ Bcl-2
homology domain” ( the third of the four such domains present in Bcl-2) and are called “BH3 –only proteins.” The sensors in turn activate
two critical (proapoptoic) effectors, Bax and Bak, which form oligomers that insert into the mitochondrial membrane and create channels
that allow proteins from the inner mitochondrial membrane to leak out into the cytoplasm.

Correct Answer. a

Copyright © 2014 Delhi Academy of Medical Sciences, All Rights Reserved. 87/94
(184). All are oncogenes except

a. C erb b2

b. N myc

c. Rb

d. CDK 4

Solution. (c) Rb
Ref: Read the text below
Sol :
- Rbis tumor suppressor gene whose main function is regulation of cell cycle Loss of Rb is associated with retinoblastomas and
osteogenic sarcomas

Correct Answer. c

(185). Desmin is a marker of tumour of

a. Epithelial origin

b. Neural origin

c. vascular origin

d. Muscle cell origin

Solution. (d) Muscle cell origin


Ref: Read the text below
Sol :
- Antibodies specific to intermediate filaments have proved to be of particular value because solid tumor cell often contain intermediate
filaments characteristic of their cell of origin,
- For example, the presence of cytokeratins, detected by immunohistochemistry, points to an epithelial origin (carcinoma) whereas
desmin is specific for neoplasms of muscle cell origin.

Correct Answer. d

(186). A 2-cm carcinoma of the breast is diagnosed by an excisional biopsy in a 36-year-old woman at 14 weeks’ gestation. The axillary nodes
are negative. The best management of this patient is to

a. Terminate the pregnancy at 16 weeks’ gestation with intra-amniotic saline

b. Induce labor at 34 weeks’ gestation, then give chemotherapy

c. Terminate the pregnancy at 36 weeks’ gestation by cesarean delivery

d. Allow the pregnancy to proceed to term and manage it expectantly

Solution. (d) Allow the pregnancy to proceed to term and manage it expectantly
Ref: Read the text below
Sol :
- The weight of evidence is that the prognosis of early-stage breast cancer is not worsened by the massive amounts of estrogen secreted
in pregnancy.
- With a small lesion and negative axillary nodes, a lumpectomy or segmental resection and subsequent expectant management is the
management of choice.
- Chemotherapy and/or radiation therapy can and should be deferred until after delivery of the fetus.
- More advanced breast cancer carries a worse prognosis and should be treated aggressively, although there is little convincing evidence
that termination of pregnancy improves the prognosis.

Correct Answer. d

Copyright © 2014 Delhi Academy of Medical Sciences, All Rights Reserved. 88/94
(187). Which of the following is the most likely predisposing factor for cholangiocarcinoma?

a. Excess alcohol intake

b. Chronic hepatitis B virus (HBV) infection

c. History of ulcerative colitis or sclerosing cholangitis

d. gallstones

Solution. (c) History of ulcerative colitis or sclerosing cholangitis


Ref: Read the text below
Sol :
- Worldwide, the presence of liver flukes (e.g., Clonorchissinensis) is the most likely predisposing factor for cholangiocarcinoma.
- Part of this increased risk is caused by the development of hepatolithiasis. The highest rate of cholangiocarcinoma is found in Southeast
Asia.
- It is thought that liver flukes and a diet high in nitrosamine are the prime reasons for this. In North America, primary sclerosing
cholangitis and chronic ulcerative colitis are the most common predisposing factors.
- Cholelithiasis, alcohol, smoking, and chronic hepatitis B are not known to be risk factors.

Correct Answer. c

(188). On colonoscopy, the most common site of villous adenomas is the :

a. Rectum

b. Descending colon

c. Sigmoid colon

d. Transverse colon

Solution. (a) Rectum


Ref: Read the text below
Sol :
- The most common site of villous adenomas is the rectum.
- The most common site of tubular adenomas is the colon

Correct Answer. a

(189). A 73-year-old man presents with fatigue and multiple lymph nodes in his neck. He has multiple large nontender lymph nodes in his neck
and axilla. The spleen is not palpable, and the remaining examination is normal. A biopsy of the lymph node reveals lymphocytic
lymphoma. Which of the following factors predicts poor prognosis?

a. No extranodal involvement

b. Poor performance status

c. Age <60

d. Low serum lactate dehydrogenase (LDH) levels

Solution. (b) Poor performance status


Ref: Read the text below
Sol :
- Prognosis of patients with non-Hodgkin’s lymphoma is best assessed with the International Prognostic Index.
- It is an index with five clinical risk factors that helps to predict the 5-year survival.
- Poor prognostic factors are age >60 years, high serum LDH level, poor performance status (either Eastern Cooperative Oncology Group
[ECOG] >2, or Karnofsky<70), Ann Arbor stage III or IV, or >1 extranodal involvement.

Correct Answer. b

Copyright © 2014 Delhi Academy of Medical Sciences, All Rights Reserved. 89/94
(190). An asymptomatic 74-year-old man has a high erythrocyte sedimentation rate (ESR) noted on routine blood work done in a yearly physical
examination. Afollow-up protein electrophoresis reveals a monoclonal immunoglobulin G (IgG) spike. Which of the statements would
suggest a plasma cell myeloma, rather than a monoclonal gammopathy of unknown significance?

a. Bence Jones protein 2 g/day

b. Normal hemoglobin

c. M component level of 2 g/dL

d. Bone marrow plasma cells of 5%

Solution. (a) Bence Jones protein 2 g/day


Ref: Read the text below
Sol :
- IgG spikes >3.5 g/dL or IgA >2 g/dL strongly suggest myeloma rather than monoclonal gammopathies of undetermined significance
(MGUS).
- MGUS is suggested when the spike is <3.5 g/dL, the marrow has fewer than 10% plasma cells, and the Bence Jones proteinuria is <1.0
g/day.
- Depressed hemoglobin levels, elevated calcium levels, progressive bone lesions, and impaired renal function suggest more advanced
stages of multiple myeloma.

Correct Answer. a

(191). Which of the following symptoms is most suggestive of an esophageal mass?

a. Liquid phase dysphagia only

b. Odynophagia with chest pain

c. Oropharyngeal dysphagia

d. Solid phase dysphagia progressing to liquid phase Dysphagia

Solution. (d) Solid phase dysphagia progressing to liquid phase Dysphagia


Ref: Read the text below
Sol :
- Although esophageal masses and cancer can lead to several types of dysphagia, the most common complaint is solid food dysphagia that
worsens to the point that liquids are also hard to swallow.
- Such a complaint warrants upper endoscopy, particularly if the patient falls in a high-risk group for esophageal cancer, with careful
examination of the stomach, trachea, and larynx.
- Odynophagia with chest pain is more reminiscent of ulcerative disease of the esophagus due to either infection, such as
cytomegalovirus or Candida, or pill esophagitis. Spasm causes severe pain as well, but this may occur independent of swallowing.
- Liquid phase dysphagia often implies a functional disorder of the esophagus rather than a mass-like obstruction. A barium swallowing
study or cine-esophagram in conjunction with a thorough history and physical examination may prove diagnostic. Oropharyngeal
dysphagia usually localizes disease quite specifically to the oropharynx.
- Early satiety is often due to gastric obstruction or extrinsic compression of the stomach (splenomegaly is a common reason for this), or
to a functional gastric disorder such as gastroparesis.

Correct Answer. d

Copyright © 2014 Delhi Academy of Medical Sciences, All Rights Reserved. 90/94
(192 Radiation exposure during infancy has been linked to which one of the
). following carcinoma ?

a. Breast

b. Melonoma

c. Thyroid

d. Lung

Solution. (c) Thyroid


Ref.:Read the text below
Sol :

Correct Answer. c

(193). MEN-II is seen with the following type of thyroid carcinoma :

a. Papillary

b. Medullary

c. Anaplastic

d. Follicular

Solution. (b) Medullary


Ref.:Robbin’s - 1222
Sol :

Correct Answer. b

Copyright © 2014 Delhi Academy of Medical Sciences, All Rights Reserved. 91/94
(194). The primary treatment for locoregionally advanced head and neck cancer consists of:

a. Surgery

b. Radiation therapy

c. Chemoradiation therapy

d. All of the above

Solution. (c) Chemoradiation therapy


Ref: Read the text below.
Sol:
- Historically, locoregionally advanced head and neck cancer (HNC) was treated with surgery (with or without adjuvant radiotherapy) or
radiotherapy alone.
- Only a minority of patients with locoregionally advanced disease can undergo adequate surgical resection, however, and the outcomes
were poor with respect to survival and organ preservation.
- Furthermore, radiotherapy alone is not sufficient to successfully treat most HNC at intermediate or advanced stages.
- Recently, chemoradiotherapy was shown to markedly improve survival and organ preservation.

Correct Answer. c

(195). Which of the following targeted agents is not a tyrosine kinase inhibitor that influences the human epidermal growth factor receptor
(HER) family signaling pathway by binding to the intracellular domain of the receptors?

a. Gefitinib (HER1 inhibitor)

b. Erlotinib (HER1 inhibitor)

c. Trastuzumab (HER2 inhibitor)

d. Lapatinib (HER1/HER2 inhibitor)

Solution. (c) Trastuzumab (HER2 inhibitor)


Ref: Read the text below.
Sol:
- Tyrosine kinase inhibitors, such as gefitinib and erlotinib (HER1 inhibitors) and lapatinib (a HER1/HER2 inhibitor), influence the
signaling pathway by interacting with the intracellular domain of the receptors of HER1 and HER2.
- Trastuzumab is a humanized monoclonal antibody that targets the extracellular domain of the HER2 receptor to prevent dimerization
with other HER family members and subsequent downstream signaling events.

Correct Answer. c

(196). Standard treatment for locally advanced epithelial ovarian cancer includes all of the following, except:

a. Debulking surgery

b. Radiation therapy

c. Intravenous chemotherapy

d. Intraperitoneal chemotherapy

Solution. (b) Radiation therapy


Ref: Read the text below.
Sol:
- Surgery and standard intravenous chemotherapy with a platinum-taxane combination induce complete remission in most patients with
newly diagnosed ovarian cancer.
- However, most patients eventually relapse and die from their disease.
- Interest is growing in the intraperitoneal delivery of chemotherapy to patients with ovarian cancer who have minimal residual disease
following initial cytoreductive surgery.
- Three randomized trials demonstrated an improvement in overall survival with intraperitoneal cisplatin compared with intravenous
chemotherapy alone.

Correct Answer. b

Copyright © 2014 Delhi Academy of Medical Sciences, All Rights Reserved. 92/94
(197). Rituximab in combination with CHOP (R-CHOP) was approved for treatment of diffuse large B-cell lymphoma (DLBCL).The addition of
rituximab to standard chemotherapy has been shown to benefit which patient population?

a. Older patients

b. Low-risk International Prognostic Index (IPI) patients

c. High-risk IPI patients

d. All of the above

Solution. (d) All of the above


Ref: Read the text below.
Sol:
- Age is an important issue in the treatment of patients with DLBCL because of the greater risk for comorbid conditions.
- Therefore, strategies were initiated to use the IPI risk factors and the age-adjusted IPI in conjunction with stage, serum lactate
dehydrogenase, and performance score as stratification factors to design therapeutic trials and appropriately select patients.
- These strategies were followed in the landmark R-CHOP trials, including the Grouped'Etude des Lymphomes de l'Adulte (GELA) and the
US Intergroup Eastern Cooperative Oncology Group (ECOG)4494/Cancer and Leukemia Group B (CALGB) 9793 trials. Results have
shown that R-CHOP is effective in both older and younger patients and in low- and high-risk IPI groups.

Correct Answer. d

(198). The immunomodulating monoclonal antibody alemtuzumab is associated with which infectious complication?

a. Cytomegalovirus (CMV) infection

b. Pneumocystis (carinii) jiroveci pneumonia (PCP)

c. Invasive fungal infection

d. All of the above

Solution. (d) All of the above


Ref: Read the text below.
Sol:
- Alemtuzumab is a humanized monoclonal antibody directed against CD52, a surface antigen expressed in high levels by B-cell chronic
lymphocytic leukemia (B-CLL) and T-prolymphocytic leukemia cells.
- Alemtuzumab has been shown to destroy target cells through antibody-dependent cellular cytotoxicity, complement-mediated cytolysis,
and induction of apoptosis.
- CD4 and CD8 T-cell counts reach their nadir approximately 4 weeks after initiation of alemtuzumab.
- High rates of standard and opportunistic infections, such as PCP, invasive aspergillosis, candidiasis, disseminated varicella-zoster virus,
mycobacterial infection, and CMV, and extremely delayed recovery of functional immune status, have been reported with alemtuzumab
use in salvage therapy of refractory or relapsed CLL.

Correct Answer. d

(199). Which characteristic is least likely for never-smokers who develop non-small cell lung cancer (NSCLC), compared with smokers who
develop NSCLC?

a. Young

b. Female

c. Better survival rate

d. Worse survival rate

Solution. (d) Worse survival rate


Ref: Read the text below.
Sol:
- Never-smokers in whom NSCLC develops are more likely to be young, be female, and have poorly differentiated tumors with higher
maximum standardized uptake values on PET scans.
- Never-smokers with early-stage cancer have a significantly better survival rate than smokers.
- Patients with a smoking history of 20 or more pack-years have worse survival. Thus, smoking not only causes lung cancer; once NSCLC
is diagnosed it worsens the prognosis.
- A biological, hormonal, and genetic explanation is currently lacking to explain these findings.

Correct Answer. d

Copyright © 2014 Delhi Academy of Medical Sciences, All Rights Reserved. 93/94
(200). What percentage of all breast cancer cases is related to genetic susceptibility?

a. 5%

b. 10%

c. 25%

d. 40%

Solution. (c) 25%


Ref: Read the text below.
Sol:
- Genetic susceptibility to breast cancer accounts for approximately 25% of all breast cancer cases. In familial breast cancer, mutations
in the BRCA1, BRCA2, CHEK2, TP53, and PTEN genes account for approximately 5% to 10% of breast and ovarian cancer cases overall.
- The prevalence of BRCA1 or BRCA2 mutations varies considerably among ethnic groups and geographical areas

Correct Answer. c

Test Answer
1.(d) 2.(a) 3.(d) 4.(a) 5.(c) 6.(b) 7.(b) 8.(d) 9.(d) 10.(a)

11.(d) 12.(d) 13.(c) 14.(c) 15.(c) 16.(b) 17.(c) 18.(d) 19.(c) 20.(d)

21.(d) 22.(c) 23.(d) 24.(d) 25.(a) 26.(d) 27.(a) 28.(c) 29.(d) 30.(c)

31.(d) 32.(d) 33.(c) 34.(c) 35.(a) 36.(a) 37.(c) 38.(b) 39.(a) 40.(c)

41.(b) 42.(d) 43.(d) 44.(c) 45.(b) 46.(b) 47.(d) 48.(d) 49.(d) 50.(c)

51.(d) 52.(b) 53.(a) 54.(d) 55.(c) 56.(d) 57.(a) 58.(a) 59.(c) 60.(d)

61.(c) 62.(d) 63.(a) 64.(d) 65.(c) 66.(d) 67.(c) 68.(d) 69.(d) 70.(d)

71.(a) 72.(b) 73.(c) 74.(d) 75.(a) 76.(d) 77.(c) 78.(b) 79.(b) 80.(c)

81.(b) 82.(b) 83.(c) 84.(a) 85.(b) 86.(d) 87.(b) 88.(d) 89.(d) 90.(c)

91.(d) 92.(c) 93.(c) 94.(b) 95.(b) 96.(d) 97.(c) 98.(d) 99.(c) 100.(a)

101.(c) 102.(c) 103.(a) 104.(d) 105.(c) 106.(c) 107.(b) 108.(b) 109.(d) 110.(d)

111.(a) 112.(a) 113.(c) 114.(c) 115.(a) 116.(a) 117.(c) 118.(d) 119.(d) 120.(d)

121.(b) 122.(d) 123.(a) 124.(c) 125.(d) 126.(d) 127.(b) 128.(c) 129.(b) 130.(c)

131.(b) 132.(a) 133.(a) 134.(a) 135.(d) 136.(b) 137.(d) 138.(c) 139.(c) 140.(c)

141.(a) 142.(c) 143.(a) 144.(a) 145.(d) 146.(d) 147.(d) 148.(d) 149.(a) 150.(b)

151.(a) 152.(a) 153.(a) 154.(d) 155.(d) 156.(a) 157.(d) 158.(a) 159.(c) 160.(a)

161.(d) 162.(b) 163.(c) 164.(c) 165.(c) 166.(a) 167.(d) 168.(a) 169.(b) 170.(c)

171.(b) 172.(b) 173.(a) 174.(b) 175.(d) 176.(c) 177.(a) 178.(b) 179.(a) 180.(d)

181.(c) 182.(b) 183.(a) 184.(c) 185.(d) 186.(d) 187.(c) 188.(a) 189.(b) 190.(a)

191.(d) 192.(c) 193.(b) 194.(c) 195.(c) 196.(b) 197.(d) 198.(d) 199.(d) 200.(c)

Copyright © 2014 Delhi Academy of Medical Sciences, All Rights Reserved. 94/94

Potrebbero piacerti anche